Indian Yearbook Through Mcqs

You might also like

Download as pdf or txt
Download as pdf or txt
You are on page 1of 64

India year book 2018

Gist
Through mcq’s

LFB IAS Join Our Telegram @ lfbias


8888 024 021 Page 1
Q.1. Consider the following c) All are correct
Statements regarding desert region d) 1 & 3 only
of India:
Q.3. Consider the following
Statements regarding National
1. The desert region can be
divided into two parts—the Flag:
‗great desert‘ and the ‗little
1. The ratio of the length to the
desert‘.
height (width) of the Flag
2. The great desert extends from
shall be 5:4.
the edge of the Rann of Kutch
2. The design of the National
beyond the Luni river
Flag was adopted by the
northward while little desert
Constituent Assembly of
extends from the Luni India on July 22, 1947.
between Jaisalmer and
Jodhpur up to the northern Identify the correct statements:
west. a) 1 only b) 2 only
c) Both d) None
Identify the correct statements:
a) 1 only b) 2 only Q.4. Consider the following
c) All are correct d) None Statements regarding State Emblem
of India:

1. It is an adaptation from the


Q.2. Consider the following
Sarnath Lion Capital of
Statements regarding Brahmputra
Asoka.
River:
2. The motto ―SatyamevaJayate‖-
1. It enters India under the Truth alone triumphs-written
name of Dihang. in Devanagari script below the
2. Near Passighat, the Debang profile of the Lion Capital is
and Lohit join the river part of the State Emblem of
Brahmaputra and the India.
combined river runs all along
the Assam valley. Identify the correct statements:
3. The principal tributaries of a) 1 only b) 2 only
Brahmaputra in India are the c) All are correct d) None
Subansiri, JiaBhareli,
Dhansiri, Puthimari, Q.5. Consider the following
Pagladiya and the Manas. Statements regarding National
Anthem of India:
Identify the correct statements:
a) 1 & 2 only 1. It was first sung on December
b) 2 & 3 only 27, 1911 at the Calcutta

LFB IAS Join Our Telegram @ lfbias


8888 024 021 Page 2
Session of the Indian National Identify the correct statements:
Congress. a) 1 only b) 2 only
2. The playing time of the full c) All are correct d) None
version of the National Anthem
is approximately 52 seconds. Q.8. Which of the following are
functions of Central Water
Identify the correct statements:
Commission (CWC)
a) 1 only b) 2 only
c) All are correct d) None 1. hydro-meteorological
observations
Q.6. Consider the following 2. monitoring water quality
Statements regarding National Song 3. techno-economic appraisal of
of India: irrigation, flood control and
multipurpose projects
1. It has a lower status with
4. monitoring glacial lakes/water
respect to Jana-gana-mana.
bodies in the Himalayan
2. The song Vande Mataram is
Region
national song of India and
was composed in Sanskrit by Choose the correct options
Bankim Chandra Chatterji.
A. 1,2 and 3
Identify the correct statements: B. 2, 3 and 4
a) 1 only b) 2 only C. 1,2 and 4
c) All are correct d) None D. All of the above

Q.7.Consider the following


Statements regarding National
Q.9. Consider the following
Calendar:
Statements regarding National
1. The National Calendar is Institute of Hydrology:
based on the Saka Era, with
1. It is a premier R&D institute
Chaitra as its first month
in the country to undertake,
with 1
aid, promote and coordinate
Chaitra falling on March 22
basic, applied and strategic
normally.
research on all aspects of
2. Dates of the National
hydrology.
Calendar have a permanent
2. The Institute has its
correspondence with dates of
headquarters at Kochi,
the Gregorian calendar,
Kerala.
which is used along with
national calendar for official Identify the correct statements:
purposes. a) 1 only b) 2 only

LFB IAS Join Our Telegram @ lfbias


8888 024 021 Page 3
c) All are correct d) None Scheduled Castes and
Scheduled Tribes.
Q.10. Consider the following
Statements regarding Civil Aviation: Identify the correct statements:
a) 1 only b) 2 only
1. The Ministry of Civil Aviation, c) All are correct d) None
nodal ministry for civil
aviation, also administratively Q.12. Consider the following
controls the institution of Statements regarding Scheduled
Commission of Railway Safety, Areas and Tribal Areas:
which is responsible for the
safety in rail travel. 1. The Fifth Schedule under
Article 244 (2) of the
2. The Airports Economic Constitution relates to those
Regulatory Authority (AERA) areas in the states of Assam,
is a constitutional body Meghalaya, Tripura and
3. Bureau of Civil Aviation Mizoram which are declared
Security (BCAS) has as ―Tribal Areas‖ and provides
responsibility to coordinate, for District Councils and/or
monitor, inspect and train Regional Councils for such
personnel in Civil Aviation Areas.
Security matters. 2. The Sixth Schedule under
Identify the correct statements: Article 244 (1) of Constitution
a) 1 & 2 only defines ―Scheduled Areas‖ as
such areas as the President
b) 2 & 3 only
may by Order declare to be
c) All are correct
Scheduled Areas after
d)1 & 3 only consultation with the
Q.11. Consider the following Governor of the state.
Statements regarding Identify the correct statements:
a) 1 only b) 2 only
1. Protection of Civil Rights Act
c) All are correct d) None
extends to the whole of India
and provides punishment for Q.13. Consider the following
the practice of untouchability. Statements regarding Position of
2. Scheduled Castes and the Governor w.r.t. scheduled areas:
Scheduled Tribes (Preventions
of Atrocities) Act aims at 1. Governor may prohibit or
preventing commission of restrict transfer of land,
offences by persons (including regulate the business of
Scheduled Castes and money lending w.r.t. tribal
Scheduled Tribes) against people or may in this regard

LFB IAS Join Our Telegram @ lfbias


8888 024 021 Page 4
repeal or amend any Act of proportion to the ST
Parliament or of the population of each State or UT
Legislature of the State.
Identify the correct statements:
2. Governor may by public a) 1 only b) 2 only
notification direct that any c) All are correct d) None
particular Act of Parliament or
State Legislature shall not Q.15. Consider the following
apply to a Scheduled Area Statements regarding Integrated
shall apply with modifications. Child Development Services (ICDS)
Scheme:
3. Governor shall make a report
to the President regarding the 1. The programme aims at
administration of the addressing health, nutrition
Scheduled Areas in that State and the development needs of
and the executive power of the young children, pregnant and
Union shall extend to the nursing mothers.
giving of directions to the
State as to the administration 2. NIPCCD has been functioning
of the said area. as an apex institution for
training of the ICDS
Identify the correct statements: functionaries
a) 1 & 2 only
b) 2 & 3 only Identify the correct statements:
c) All are correct a) 1 only b) 2 only
d) 1 & 3 only c) All are correct d) None

Q.14.Consider the following Q.16. Consider the following


Statements regarding Tribal Sub Statements regarding Sports
Plan: awards:

1. The present Tribal Sub Plan 1. While criteria for Rajiv Gandhi
(TSP) strategy was KhelRatna Award involves
developed for rapid outstanding performance by a
socioeconomic sportsperson over a period of
development of tribal people four years, Arjuna award in
and was adopted for the first addition to good performance
time in the Eighth Five Year over past 4 years
Plan. requires qualities of
leadership, sportsmanship
2. The funds provided under the and a sense of discipline.
Tribal Sub Plan of the State
have to be at least equal in 2. With a view to recognizing the
contribution made to sports

LFB IAS Join Our Telegram @ lfbias


8888 024 021 Page 5
development by sportspersons Q.19.Consider the following
and coaches, Government has Statements regarding Heavy Water
instituted a new award titled Board:
‗Rashtriya Khel Protsahan
Puruskar‘ 1. It has contributed
successfully to the first stage
Identify the correct statements: of Nuclear Power Programme
a) 1 only b) 2 only by producing heavy water for
c) All are correct d) None all Pressurised Heavy Water
Reactors.
Q.17.__________ is an ongoing
initiative for attraction of young 2. Board is not only self-
talent for science and research. sufficient in meeting the
domestic demand of heavy
a) INSPIRE water, but is also geared up to
b) KIRAN supply heavy water for the
c) SATYAM future PHWRs and AHWRs.
d) Unnat Bharat Abhiyan Identify the correct statements:
Q.18.Consider the following a) 1 only b) 2 only
Statements regarding Council of c) All are correct d) None
Scientific & Industrial Research Q.20.Consider the following
(CSIR): Statements regarding Indian Space
1. It is an autonomous body Programme:
setup in 1942 which is known
1. Space activities in the country
for its cutting edge R&D
were initiated with the setting
knowledge base in diverse
up of Indian National
S&T areas.
Committee for Space Research
2. CSIR has created and is the (INCOSPAR) in 1962.
custodian of Traditional
2. The Space Commission was
Knowledge Digital Library
constituted and established
(TKDL) which is a powerful
the Department of Space
weapon against unethical
(DOS) in June, 1972 and
commercial exploitation of
brought ISRO under DOS in
Indian traditional knowledge.
1972.
Identify the correct statements:
Identify the correct statements:
a) 1 only b) 2 only
a) 1 only b) 2 only
c) All are correct d) None
c) All are correct d) None

LFB IAS Join Our Telegram @ lfbias


8888 024 021 Page 6
Q.21.Consider the following c) All are correct d) None
Statements regarding Press
Q.23.Consider the following
Information Bureau:
Statements regarding Commission
1. The Press Information Bureau for Agricultural Costs and Prices
(PIB) is a non-government
organisation with objective to 1. It is mandated to advice on
disseminate information to the price policy (MSP)
the print and electronic media 2. Cost of production (CoP) is the
on government policies, only factors in the
programmes, initiatives and determination of MSP
achievements. Which of the above are correct
2. PIB has a dedicated unit for
a) 1 only b)2 only
the publicity and 24*7 media
c) Both are correct d)None
support to the Prime
Minister‘s office.
Identify the correct statements: Q.24.Consider the following
a) 1 only b) 2 only Statements
c) All are correct d) None
1. PRASAD scheme has a vision
Q.22.Consider the following to develop theme based
Statements regarding Mahatma tourist circuits
Gandhi National Rural Employment 2. Swadesh Darshan scheme, 25
sites of religious significance
Guarantee Act:
have been identified for
1. It is a rights based wage development
employment programme Which of the above are correct
implemented in rural areas of
the country. a) 1 only
b) 2 only
2. It aims at enhancing
c) Both are correct
livelihood security by
d) None
providing upto one hundred
days of guaranteed wage Q.25. Consider the following
employment in calendar year Statements regarding Central
to every rural household Statistics Office
whose adult members
volunteer to do skilled manual 1. coordinates the
work. statisticalactivities in the
country
Identify the correct statements:
2. It is responsible for the
a) 1 only b) 2 only decadal sensus
LFB IAS Join Our Telegram @ lfbias
8888 024 021 Page 7
3. It assists only in development c) 1, 2 and 3 d) None
of statistics of the nation
Q.28.Consider the following
Which of the above are correct Statements
a) 1 only 1. E-Taal Database is developed
b) 1 and 2 only to send messages and emails
c) 1, 2 and 3 to public representatives
d) 1 and 3 only andgovernment employees.
Q.26.Consider the following 2. E-Sampark is a web portal for
Statements dissemination of e-
transactions statistics of
1. CSO compiles the Index of national and state level e-
Industrial Production (IIP) governance projects including
2. The Base year for the Existing mission mode projects.
Series of WPI is 2004-05.
Which of the above are correct
3. Base Year of the CPI2010
Which of the above are correct a) 1 only b) 2 only
b) Both are correct d) None
a) 1 only
Q.29.Consider the following
b) 1 and 2 only
Statements regarding Defence
c) 1, 2 and 3
d) 1 and 3 only Research and Development
Organisation (DRDO)
Q.27.Consider the following
Statements regarding The Members 1. DRDO is headed by the
of Parliament Local Area Raksha Mantri
Development Scheme (MPLADS) 2. Its mandate also includes
support national cyber
1. The MPLADS is under the security architecture
control of Ministry of Rural 3. The DRDO ensures training to
Development. all cadres of personnel
2. The funds released under the
Which of the above are correct
Scheme are lapsable
3. The role of the Members of a) 1 only
Parliament is not limited to b) 2 and 3 only
recommend works. But also c) 1, 2 and 3
sanction, execute and d) None
complete the works
Q.30.Consider the following
Which of the above are correct Statements regarding Kasturba
a) 1 only b) 1 and 2 only Gandhi BalikaVidyalaya (KGBV)

LFB IAS Join Our Telegram @ lfbias


8888 024 021 Page 8
1. KGBV are non-residential 1. The schemes target mainly the
upper primary schools forgirls senior citizens above 65 years
from SC, ST, OBC Muslim of age.
communities and BPL girls. 2. The beneficiaries receive 35 kg
2. KGBVs provide for a minimum of food grains per household
reservation of 75 per cent per month.
seats for girls from
Which of the statements given above
SC/ST/OBCand minorities
is/are correct?
communities
3. 25 per cent to girls from a) 1 only b) 2 only
families that live below the
poverty line. c) Both 1 and 2 d) Neither 1nor 2

Which of the above are correct Q.33. The Hindu Adoption and
Maintenance Act of 1956 is
a) 1 only
applicable to persons belonging to
b) 1, 2 and 3
which of the following religions?
c) 2 and 3 only
d) None 1. Hinduism
Q.31. With reference to the 2. Jainism
Sustainable Development Goals 3. Buddhism
(SDGs), consider the following 4. Sikhism
statements: Select the correct answer using the
code given below.
1.They are 17 goals with 169 targets
which aim to end all forms of (a) 1 only
poverty. (b) 1, 2 and 3 only
2. They are applicable to developing (c) 2 and 3 only
countries only. (d) 1, 2, 3 and 4
3.They are legally binding.
Q.34. Consider the following
Which of the statements given above statements about the pricing
is/are correct? mechanism of petroleum products
in India:
a) 1 only
b) 1 and 2 only 1. The prices of all petroleum
c) 2 and 3 only products are market
d) 1, 2 and 3 determined.
Q.32. Consider the following 2. The prices of petrol and diesel
statements about Antyodaya Anna are determined on import
Yojana (AAY): parity basis while that of PDS

LFB IAS Join Our Telegram @ lfbias


8888 024 021 Page 9
kerosene and domestic LPG 1. It aims to achieve a target of
on trade parity basis. grid connected solar power of
100,000 MW by 2022.
Which of the statements given above
2. It seeks to harvest 40,000 MW
is/are correct? solar energy by setting up
(a) 1 only rooftop panels.
(b) 2 only Which of the statements given above
(c) Both 1 and 2 is/are correct?
(d) Neither 1 nor 2
(a) 1 only
Q.35. In the context of government
(b) 2 only
accounts, consider the following
(c) Both 1 and 2
statements: (d) Neither 1 nor 2
1. No amount can be withdrawn Q.37. With reference to the
from the Consolidated Fund of Department of Atomic Energy's 3-
India without prior stage Nuclear Power Programme,
authorisation from consider the following statements:
Parliament.
2. Any amount withdrawn from 1. The first stage envisages
the Public Account of India setting up of Fast Breeder
requires a subsequent Reactors (FBRs).
authorisation from the 2. Plutonium is produced in the
Parliament. second stage by irradiation of
3. No parliamentary uranium-238.
authorisation for payments 3. The third stage is based on
from the Contingency Fund is the Plutonium-Uranium cycle.
required, either prior or
Which of the statements given above
subsequent to incurring such
expenditure. is/are correct?

Which of the statements given above (a) 1 and 3 only


is/are correct? (b) 2 and 3 only
(c) 2 only
(a) 1 only (d) 1, 2 and 3
(b) 1 and 3 only
Q.38. Which of the following
(c) 2 and 3 only
(d) none statements is/are correct regarding
the scheme "Ek Bharat Shreshtha
Q.36. With reference to the Bharat"?
Jawaharlal Nehru National Solar
Mission (JNNSM), consider the 1. It aims to promote the spirit of
following statements: national integration through

LFB IAS Join Our Telegram @ lfbias


8888 024 021 Page 10
engagement between all 2. Protection and assistance to
Indian States and UTs. fishermen at sea while in
2. It involves student exchange distress.
programs between the states. 3. Preservation and protection of
3. It is being jointly implemented marine environment and
by Ministry of Human prevention of marine
Resource Development and pollution.
Ministry of Culture.
Select the correct answer using the
Select the correct answer using the code give below.
code given below.
(a) 1 and 2 only
(a) 1 and 2 only (b) 1 only
(b) 2 only (c) 2 and 3 only
(c) 1 and 3 only (d) 1, 2 and 3
(d) 1, 2 and 3
Q.39. Over 10 greenfield airports Q.41. The term 'Sowa Rigpa' refers
have been recently approved by the to
government. Which of the following
a) a traditional medicine system in
correctly describes a green field
Tibet.
airport?
b) a traditional system of agriculture
(a) An airport which is completely in Manipur.
dependent on solar energy for c) an architectural style in Buddhist
power. monasteries in Ladakh.
(b) An airport which is d) an ancient form of dance drama
constructed on an in Sikkim.
undeveloped site.
(c) An airport which has at least
70% area under green cover. Q.42. With reference to Financial
(d) None of the above. Stability Board (FSB), consider the
Q.40. Indian Coast Guard, a following statements:
statutory body, is mandated with
1. It was established under the
which of the following duties under aegis of G20.
the Coast Guard Act, 1978? 2. It has been established to
develop and implement
1. Safety and protection of
policies in the interest of
artificial islands and offshore
financial stability.
terminals, installations and
3. India is one of the members of
devices in maritime zones.
FSB.

LFB IAS Join Our Telegram @ lfbias


8888 024 021 Page 11
Which of the statements given above 1. The Finance Bill, presented
is/are correct? before Parliament at the time
of presentation of the Budget,
(a) 1 and 2 only is a Money Bill.
(b) 1 and 3 only 2. The Vote on Account is
(c) 2 and 3 only obtained from Parliament
(d) 1, 2 and 3 through a Finance Bill.
Q.43. Which among the following Which of the statements given above
are the components of Pradhan is/are correct?
Mantri Krishi Sinchai Yojana?
(a) 1 only
1. Accelerated Irrigation Benefits (b) 2 only
Programme (c) Both 1 and 2
2. Har Khet Ko Pani (d) Neither 1 nor 2
3. Watershed development
4. Per drop more crop Q.46. Which of the following are
administered by the Ministry of
Select the correct answer using the Corporate Affairs (MCA)?
code given below.
1. Companies Act, 2013
(a) 1, 3 and 4 only 2. The Insolvency and
(b) 1 and 3 only Bankruptcy Code, 2016
(c) 2 and 4 only 3. India trade portal
(d) 1, 2, 3 and 4 4. The Bureau of Indian
Q.44. Which among the following Standards Act, 2016
is/are constituents of Prasar Select the correct answer using the
Bharati? code given below.

1. Doordarshan (a) 1 and 2 only


2. All India Radio (b) 1, 3 and 4 only
3. Press Council of India (c) 3 and 4 only
(d) 1, 2 and 3 only
Select the correct answer using the
code given below. Q.47. Recently, a leap second was
added to the Universal Coordinated
(a) 1 and 2 only
Time (UTC). In regard to this
(b) 1 only
consider the following statements:
(c) 2 and 3 only
(d) 1, 2 and 3 1. It is added to synchronize
Q.45. In the context of the general clocks worldwide with the
Budget, consider the following Earth's ever slowing rotation.
statements:

LFB IAS Join Our Telegram @ lfbias


8888 024 021 Page 12
2. They are always added to the (a) 1 only
leap years. (b) 2 and 3 only
(c) 1 and 3 only
Which of the statements given above
(d) 1, 2 and 3
is / are correct?
Q.50. Which of the following are
(a) 1 only central laws are implemented by the
(b) 2 only Ministry of Labour and
(c) Both 1 and 2
Employment?
(d) 1, 2 and 3
Q.48. With reference to the Central 1. Industrial Disputes Act,
1947
Ground Water Authority (CGWA),
2. Child Labour (Prohibition
consider the following statements:
and Regulation) Act, 1986
1. It aims to regulate and control 3. Maternity Benefit Act, 1961
ground water management Select the correct answer using the
and development in the code given below.
country.
2. It has been setup under the (a) 1 and 2 only
Environment (Protection) Act, (b) 2 only
1986. (c) 1 and 3 only
(d) 1, 2 and 3
Which of the statements given above
is/are correct? Q.51. The Financial Sector
Assessment Programme is a joint
(a) 1 only
initiative of:
(b) 2 only
(c) Both 1 and 2 1. World Economic Forum
(d) Neither 1 nor 2 (WEF)
2. International Monetary
Fund (IMF)
Q.49. Which among the following 3. World Bank (WB)
initiatives focuses on improvement Select the correct answer using the
of research and innovations in the code given below.
field of science and technology?
(a) 1 and 2 only
1. INSPIRE (b) 1 and 3 only
2. KIRAN (c) 2 and 3 only
3. IMPRINT India (d) 1, 2 and 3
Select the correct answer using the Q.52. Scientists have suggested
code given below. modifying mosquitoes genetically
(GM male mosquitoes) to decrease
LFB IAS Join Our Telegram @ lfbias
8888 024 021 Page 13
the incidence of vector-born Q.54. The One Rank One Pension
diseases like Dengue and Malaria. (OROP) scheme has been
In this regard, consider the following announced for
statements:
(a) Unorganised sector
1. The technique involves mating (b) Armed forces
of GM male mosquitoes with (c) Government school teachers
female carriers of the vector- (d) All India Services
borne diseases.
2. The offspring of GM male
mosquitos cannot survive Q.55. Diamond Triangle, a famous
beyond larval stage. heritage network in Orissa, is
Which of the statements given related to
above is/are correct?
(a) Vajrayana sect of Buddhism
(a) 1only (b) Santhal tribal culture
(b) 2 only (c) Bhakti and Sufi traditions
(c) Both 1 and 2 (d) Ashokan inscriptions and
(d) Neither 1 nor 2 pillars

Q.53. Consider the following


statements regarding the Minimum
Q.56. The objective of Padhe
Wages Act, 1948:
Bharat, Badhe Bharat is
1. It was enacted to safeguard the
(a) to ensure vocational
interests of the workers in the
education to graduates.
organised sector only.
(b) to ensure quality at the
2. Both the Central and state
foundational years of
governments can fix and enforce
schooling.
the payment of minimum wages.
(c) to promote old age education.
3. The minimum wages are (d) to promote female literacy .
adjusted for inflation based on
the Consumer Price Index. Q.57. Consider the following
statements with reference to 'SABLA'
Which of the statements given above
scheme:
is/are correct?
1. It aims at all-round
(a) 1 and 3 only
development of adolescent
(b) 1 only
girls of 11-18 years of age.
(c) 2 and 3 only
2. It is being implemented using
(d) 1, 2 and 3
the platform of Integrated
Child Development Scheme

LFB IAS Join Our Telegram @ lfbias


8888 024 021 Page 14
(ICDS) through Aanganwadi Q.60. Which of the following is/are
Centres. the tributaries of river Ganga?
3. It is an initiative of the
Ministry of Women and Child 1. Sone
Development. 2. Gandak
3. Betwa
Which of the statements given above
is/are correct? Select the correct answer using the
code given below.
(a) 1 only
(b) 1 and 2 only (a) 1 only
(c) 2 and 3 only (b) 1 and 2 only
(d) 1, 2 and 3 (c) 2 and 3 only
(d) 1, 2 and 3
Q.58. With reference to the Polar
Cyclone, consider the following Q.61. Which of the following
statements: countries are members of both Gulf
Cooperation Council (GCC) and
1. It is a large area of low Organization of the Petroleum
pressure and cold air Exporting Countries (OPEC)?
surrounding both the poles of
the earth. 1. Iran
2. It is stronger in summer 2. Bahrain
months than in winter 3. Kuwait
months. 4. Qatar
3. Its formation results in ozone
Select the correct answer using the
depletion.
code given below.
Which of the statements given above
is/are correct? (a) 1, 3 and 4 only
(b) 3 and 4 only
(a) 1 only (c) 2 and 4 only
(b) 1 and 3 only (d) 1, 2 and 3 only
(c) 2 and 3 only
Q.62. Recently, the Supreme Court
(d) 1, 2 and 3
refused a plea to frame a national
Q.59. Which of the following policy on National Symbols. Which
country is not the member of of the following National Symbols
MERCOSUR? is/are mentioned under Article 51A
of the Indian constitution?
(a) Brazil
(b) Argentina 1. National Flag
(c) Chile 2. National Song
(d) Uruguay 3. National Emblem

LFB IAS Join Our Telegram @ lfbias


8888 024 021 Page 15
Select the correct answer using the Select the correct answer using the
code given below. code given below.

(a) 1 only (a) 3 only


(b) 1 and 2 only (b) 1 and 2 only
(c) 2 and 3 only (c) 2 and 3 only
(d) 1, 2 and 3 (d) 1, 2 and 3
Q.63. Consider the following Q.65. Consider the following
statements regarding the National statements regarding University
Child Labour Project (NCLP) Grant Commission(UGC):
Scheme:
1. It is a statutory body.
1. It is under the Ministry of 2. It serves as link between state
Women and Child and union governments and
Development. higher education institutions.
2. Projects under the scheme 3. It is the only grant giving body
are entirely funded by the in country to regulate
Central Government. standards in higher
3. It aims to rehabilitate education.
children working in
Which of the statements given above
hazardous occupations.
is / are correct?
Which of the statements given above
are correct? (a) 1 and 3 only
(b) 2 and 3 only
(a) 1 and 3 only (c) 1 and 2 only
(b) 2 and 3 only (d) 1, 2 and 3
(c) 1 and 2 only
Q.66. skyTran, recently in news, is
(d) 1, 2 and 3
Q.64. Which of the following (a) a high speed, elevated
Personal Rapid Transit
statements is/are correct regarding
system.
the Botanical Survey of India (BSI)?
(b) a system of control for solar
1. It works under the Ministry of power facilities and the
Environment, Forest and monitoring of photovoltaic
Climate Change. plants.
2. It brings out an inventory of (c) a two-stage rocket designed
endangered plants in the form and manufactured by SpaceX.
of a publication titled 'Red (d) an unmanned aircraft system
Data Book'. developed by DRDO.
3. Its headquarter is located at
Kolkata.

LFB IAS Join Our Telegram @ lfbias


8888 024 021 Page 16
Q.67. With reference to Cassini- (a) 1, 2 and 3 only
Huygens spacecraft, consider the (b) 2 and 3 only
following statements: (c) 1 and 4 only
(d) 1, 2, 3 and 4
1. It is an unmanned spacecraft
sent to the planet Saturn.
2. It is a joint endeavour of NASA Q.70. With reference to the
and ISRO. Securities Appellate Tribunal (SAT),
consider the following statements:
Which of the statements given above
is/are correct? 1. It is a statutory body.
2. It is bound by procedure laid
(a) 1 only down by Code of Civil
(b) 2 only Procedure.
(c) Both 1 and 2 3. Any person aggrieved by a
(d) Neither 1 nor 2 decision/order of SAT may file
Q.68. Which among the following an appeal to Supreme Court.
are part of National Waterway 4? Which of the statements given above
is/are correct?
1. River Godavari
2. River Krishna (a) 1 and 2 only
3. Buckingham canal (b) 1 and 3 only
4. East coast canal (c) 2 and 3 only
Select the correct answer using the (d) 1, 2 and 3
code given below. Q.71. Consider the following pairs:

(a) 2 and 3 only Ancient Names Present Day


(b) 1 and 4 only States
(c) 1, 2 and 3 only 1. Dakshin Kaushal: Chhatisgarh
(d) 1, 2, 3 and 4
2. Pragjyotish: Assam
Q.69. Which among the following
animals are depicted in the State 3. Gopakapattam: Goa
Emblem of India?
4. Kalinga: Bihar
1. Lion
2. Bull Which of the pairs given above are
3. Horse correctly matched?
4. Elephant
(a) 1, 2 and 3 only
Select the correct answer using the (b) 1, 3 and 4 only
code given below. (c) 2 and 4 only
(d) 1 and 2 only

LFB IAS Join Our Telegram @ lfbias


8888 024 021 Page 17
Q.72. With reference to River Project
International Fund for Agricultural
1. Satluj : Bhakra Dam
Development (IFAD), consider the
following statements: 2. Beas : Pong Dam
1. It is a specialized agency of 3. Ravi : Ranjit Sagar Dam
the United Nations.
2. It is dedicated to eradicating Which of the pairs given above
poverty and hunger in rural is/are correctly matched?
areas of developing countries.
3. India is a founding member of (a) 1 only
IFAD. (b) 1 and 2 only
(c) 2 and 3 only
Which of the statements given above (d) 1, 2 and 3
is/are correct?
Q.75. Which of the following is/are
(a) 1 and 2 only part of India's foreign exchange
(b) 1 and 3 only reserves?
(c) 2 and 3 only
(d) 1, 2 and 3 1. Gold
2. Foreign Currency Assets
Q.73. With reference to the Central (FCAs)
Statistics Office (CSO), consider the 3. Special Drawing Rights
following statements: (SDRs)
1. It is a under the Ministry of Select the correct answer using the
Finance. code given below.
2. It releases Consumer Price
Indices (CPI) for all-India as (a) 1 and 2 only
well as states/union (b) 1 and 3 only
territories. (c) 2 and 3 only
3. It uses 2010 as the base year (d) 1, 2 and 3
for calculation of CPI. Q.76. Which of the following pairs of
Which of the statements given above Departments and Ministries of the
is/are correct? Government of India are not
correctly matched?
(a) 1 only
(b) 1 and 3 only 1. Department of Official
(c) 2 only Language - Ministry of
(d) 1, 2 and 3 Culture
2. Department of Space -
Q.74. Consider the following pairs:
Ministry of Science and
Technology

LFB IAS Join Our Telegram @ lfbias


8888 024 021 Page 18
3. Department of Disinvestment Which of the pairs given above
– Ministry of Commerce and is/are correctly matched?
Industry
4. Department of Land (a) 2 and 3 only
Resources - Ministry of (b) 1 and 2 only
Agriculture (c) 3 only
(d) 1, 2 and 3
Select the correct answer using the
code given below. Q.79. Consider the following
statements regarding Central
(a) 1 and 2 only Adoption Resource Agency (CARA):
(b) 3 and 4 only
(c) 1, 2 and 4 only 1. It regulates both in-country
(d) 1, 2, 3 and 4 and intercountry adoption in
India.
Q.77. Which of the following are the 2. It became a statutory body
indigenously developed Unmanned under Juvenile Justice Act,
Aerial Vehicle (UAV) in India? 2015.
3. It works under the Ministry of
1. NISHANT
Women and Child
2. TAPAS 201 (RUSTOM-II)
Development.
3. AURA
4. LAKSHYA Which of the statements given above
is/are correct?
Select the correct answer using the
code given below. (a) 1 and 2 only
(b) 2 only
(a) 1 and 4 only
(c) 2 and 3 only
(b) 2 and 3 only
(d) 1, 2 and 3
(c) 2, 3 and 4 only
(d) 1, 2, 3 and 4 Q.80. With reference to Kisan Credit
Card Scheme, consider the following
statements:
Q.78. With respect to the mineral
1. The card provided under the
resources in India, consider the
scheme is an ATM enabled
following pairs:
debit card.
State Mineral 2. It aims to provide interest free
short term loans to farmers.
1. Rajasthan : Crude oil 3. It is operational throughout
the country except for the
2. Meghalaya : Uranium state of Jammu and Kashmir.
3. Jharkhand : Coal

LFB IAS Join Our Telegram @ lfbias


8888 024 021 Page 19
Which of the statements given above
is/are correct?
Q.83. Which among the following
(a) 1 only organisation is responsible for
(b) 2 and 3 only deciding the nomenclature of
(c) 1 and 3 only elements of the periodic table?
(d) 1, 2 and 3
(a) International Federation of
Q.81. Consider the following Clinical Chemistry and
organisations under the Ministry of Laboratory Medicine
Culture: (b) International Organization for
Chemical Sciences in
1. Lalit Kala Academy
Development
2. National School of Drama
(c) International Union of Pure
3. Kalakshetra Foundation
and Applied Chemistry
4. Archaeological Survey of India
(d) International Union of
Which of the above organisations Biochemistry and Molecular
are related to preservation, Biology
protection and promotion of Q.84. With regard to fertilizer sector
intangible cultural heritage in India? in India,consider the following
(a) 1 and 3 only statements:
(b) 1, 2 and 3 only 1. Urea is the only fertilizer
(c) 2 and 4 only under statutory price control.
(d) 1, 2, 3 and 4 2. Import of all fertilizers is
Q.82. Consider the following restricted and permitted only
statements regarding biofuels: through the State Trading
Enterprises (STEs).
1. Both first generation biofuels
Which of the statements given above
and second generation
biofuels are produced from is/are correct?
non- food crops only. (a) 1 only
2. Burning of biofuels does not (b) 2 only
emit greenhouse gases. (c) Both 1 and 2
Which of the statements given above (d) Neither 1 nor 2
is / are correct? Q.85. Jal Kranthi Abhiyan launched
(a) 1 only by the central governments aims to:
(b) 2 only (a) Strengthen involvement of
(c) Both 1 and 2 Panchayati Raj Institutions and
(d) Neither 1 nor 2 local bodies in the water

LFB IAS Join Our Telegram @ lfbias


8888 024 021 Page 20
security and development Outlook : UNEP
schemes.
(b) Transfer water from water Which of the pairs given above
surplus basins to water deficit is/are correctly matched?
basins through inter basin
(a) 1 and 2 only
water transfer projects.
(b) 1 only
(c) Provide subsidy to farmers who
(c) 2 and 3 only
are implementing rainwater
(d) 1, 2 and 3
harvesting system in their
irrigation wells. Q.88. Which among the following is
(d) Develop the villages located the only riverine major port in
along the river Ganga which India?
have historic, cultural, and
religious and/or tourist (a) Kolkata Port
importance. (b) Paradip port
(c) Cochin port
Q.86. Consider the following
(d) Mangalore port
statements regarding Agni V missile:
Q.89. Electronic Bank Realization
1. It is a nuclear capable cruise Certificate (e- BRC) is related to:
missile.
2. It can deliver nuclear (a) Open Market Operations of
warheads about 7500 km the RBI
away. (b) foreign trade
3. It can be launched from (c) public borrowing by
mobile platforms. government
(d) asset reconstruction
Which of the statements given above
is/are correct? Q.90. Which among the following
pairs is/are correctly matched with
(a) 1 and 2 only respect to nanoparticles and their
(b) 1 and 3 only
applications?
(c) 3 only
(d) 1, 2 and 3 Nanoparticles Application
Q.87. Consider the following pairs: 1. Copper nanoparticles :
Reports Published by Antimicrobial films

1. World Investment 2. Silver nanoparticles : Air


Report : UNCTAD conditioners
2. Global Wage Report : ILO 3. Gold nanoparticles : Disease
3. Global Environment diagnosis

LFB IAS Join Our Telegram @ lfbias


8888 024 021 Page 21
Select the correct answer using the (b) 2 and 4 only
code given below. (c) 2, 3 and 4 only
(d) 1, 2, 3 and 4
(a) 3 only
(b) 2 only
(c) 2 and 3 only
Q.93. Which of the following
(d) 1, 2 and 3
statements regarding Press
Q.91. With reference to pulses Information Bureau (PIB) are
cultivation in India, consider the correct?
following statements:
1. It has a dedicated unit for the
1. India is the largest producer as publicity and media support
well as the consumer of pulses to the Prime Minister's office.
in the world. 2. It provides accreditation to
2. Pulses have been covered domestic as well as foreign
under the National Food media.
Security Mission. 3. It provides feedback to the
3. N.K. Singh committee was Government regarding its
setup to tackle the shortage of policies.
pulses in India.
Select the correct answer using the
Which of the statements given above code given below.
is/are correct?
(a) 1 and 2 only
(a) 1 only (b) 1 and 3 only
(b) 1 and 2 only (c) 2 and 3 only
(c) 2 and 3 only (d) 1, 2 and 3
(d) 1, 2 and 3
Q.94. The objective of 'SAMVAY' is
Q.92. Consider the following pairs:
(a) to promote communication
Mangrove Site State between state and union
governments.
1. Pulicat: Odisha (b) to provide platform for
industry to interact with
2. Karwar: Karnataka
Union government.
3. Bhaitarkanika: Tamil Nadu (c) to provide mobility between
vocational and academic
4. Dumas-Ubhrat: Gujarat education.
(d) to enable digital transactions
Which of the pairs given above at panchayat levels.
is/are correctly matched?

(a) 1 and 3 only


LFB IAS Join Our Telegram @ lfbias
8888 024 021 Page 22
Q.95. ShinMaywa US 2i, recently (a) 3 only
seen in news, is (b) 2 and 3 only
(c) 1 and 2 only
(a) Amphibious aircraft developed (d) 1 and 3 only
by Japan.
(b) Remote sensing satellite Q.98. Consider the following pairs:
launched by China.
Islands in the Indian Ocean
(c) Fastest bullet train developed
Region Country
by Japan.
(d) Joint naval exercise between 1. Reunion France
USA and Japan.
2. Cocos UK
Q.96. Which of the following is
correct about Universal Account 3. Christmas Australia
Number?
Which of the pairs given above
(a) It provides automatic portability is/are correctly matched?
of Provident Fund account on
change of employment. (a) 1 and 2 only
(b) It is the universal identification (b) 3 only
key of the income tax payer to (c) 1 and 3 only
track his financial transactions. (d) 1, 2 and 3
(c) It is the single account number
that is universally used across
the banks. Q.99. Which of the following is the
(d) It is the account used by the first passenger drone developed
beneficiaries of public recently?
distribution system.
(a) Pawan
Q.97. Consider the following
(b) Ehang 184
statements with regard to recently
(c) Aerostar
launched Resourcesat-2A satellite (d) Mirach 26
by ISRO:
Q.100. GRAPES-3, an Indo-
1. It is a remote sensing satellite. Japanese collaboration, aims to
2. It will be used for
telecommunications purposes. (a) Study cosmic rays.
3. It was placed into Sun (b) Develop a new genetic variety
Synchronous Orbit. of grapes.
(c) Develop methods to clean
Which of the statements given above space pollution.
is / are correct? (d) Explore deep-sea minerals.

LFB IAS Join Our Telegram @ lfbias


8888 024 021 Page 23
Answers & Explanation
Ans 1: C
The desert region can be divided into two parts—the ‗great desert‘ and the ‗little
desert‘. The great desert extends from the edge of the Rann of Kutch beyond
the Luni River northward. The whole of the Rajasthan-Sind frontier runs
through this. The little desert extends from the Luni between Jaisalmer and
Jodhpur up to the northern west. Between the great and the little deserts lies a
zone of absolutely sterile country, consisting of rocky land, cut up by limestone
ridges.
Ans 2: C
The principal tributaries of Brahmaputra in India are the Subansiri, JiaBhareli,
Dhansiri,Puthimari, Pagladiya and the Manas. The Brahmaputra in
Bangladesh fed by Teesta, etc. finally falls into the Ganga. The Barak River, the
head stream of Meghna, rises in the hills in Manipur. The important tributaries
of the river are Makku, Trang, Tuivai, Jiri, Sonai, Rukni, Katakhal, Dhaleswari,
Langachini, Maduva and Jatinga. Barak continues in Bangladesh till the
combined Ganga-Brahmaputra join it near Bhairab Bazar.
Ans 3: B
THE National Flag shall be a tricolour panel made up of three rectangular
panels or sub-panels of equal width. The colour of the top panel shall be India
saffron (kesari) and that of the bottom panel India green. The middle panel
shall be white, bearing at its centre the design of Ashoka Chakra innavy blue
colour with 24 equally spaced spokes. The Ashoka Chakra shall preferably be
screen printed or otherwise printed or stenciled or suitably embroidered and
shall be completely visible on both sides of the Flag in the centre of the white
panel. The National Flag shall be rectangular in shape. The ratio of the length
to the height (width) of the Flag shall be 3:2. The design of the National
Flag was adopted by the Constituent Assembly of India on July 22, 1947.Apart
from non-statutory instructions issued by the government from
Ans 4: C
The State Emblem of India is an adaptation of the Lion Capital of Asoka at
Sarnath. In the original, the Lion Capital has four lions mounted back to back,
on a circular abacus. The frieze of the abacus is adorned with sculptures in

LFB IAS Join Our Telegram @ lfbias


8888 024 021 Page 24
high relief of an elephant, a galloping horse, a bull and a lion separated by
intervening Dharma Chakras. The abacus rests on a bell shaped lotus.
The profile of the Lion Capital showing three lions mounted on the abacus with
a Dharma Chakra in the centre, a bull on the right and a galloping horse on
the left, and outlines of Dharma Chakras on the extreme right and left was
adopted as the State Emblem of India on January 26, 1950. The bell shaped
lotus was omitted. The motto ―SatyamevaJayate”-Truth alone triumphs—
written in Devanagari script below the profile of the Lion Capital is part of the
State Emblem of India.
In the State Emblem lies the official seal of the Government of India. Its use is
regulated by the State Emblem of India (Prohibition of Improper Use) Act, 2005
and The State Emblem of India(Regulation of Use) Rules, 2007 [read with State
Emblem of India (Regulation of Use) Amendment Rules, 2010].
Ans 5. C
The song Jana-gana-mana, composed originally in Bangla by Rabindranath
Tagore, was adopted in its Hindi version by the Constituent Assembly as the
National Anthem of India on January 24, 1950. It was first sung on December
27, 1911 at the Calcutta Session of the Indian National Congress. The complete
song consists of five stanzas. The first stanza contains the full version of the
National Anthem
The playing time of the full version of the National Anthem is approximately 52
seconds. A short version consisting of the first and last lines of the National
Anthem (playing time approximately 20seconds) is also played on certain
occasions.
Ans 6: B
The song VandeMataram, composed in Sanskrit by BankimchandraChatterji,
was a source of inspiration to the people in their struggle for freedom. It has an
equal status with Jana-gana-mana.
The first political occasion when it was sung was the 1896 session of the
Indian National Congress.
Ans 7. C
The National Calendar based on the Saka Era, with Chaitraas its first month
and a normal year of365 days was adopted from March 22, 1957 along with the
Gregorian calendar for the following official purposes : (i) Gazette of India, (ii)
news broadcast by All India Radio, (iii) calendars issued by the Government of
India and (iv) Government communications addressed to the public.

LFB IAS Join Our Telegram @ lfbias


8888 024 021 Page 25
Dates of the National Calendar have a permanent correspondence with dates of
the Gregorian Calendar, 1 Chaitra falling on March 22 normally and on March
21 in leap year.
Ans 8: D
Central Water Commission (CWC) is divided among 3 wings namely, designs
and research wing (D&R), water planning and projects wing (WP&P) and river
management wing (RM).
Functions
1. Hydrological Observations
2. Water Quality Monitoring
3. Flood Forecasting
4. Project Appraisal
5. Project Monitoring
6. Monitoring of Glacial Lakes in the Himalayan Regions
7. Design Consultancy
8. Dam Rehabilitation and Improvement Project
9. National Register of Large Dams
10. Water Resource Information System
11. Training and Capacity Building

Ans 9: A
The National Institute of Hydrology (NIH), established in 1978 as an
autonomous organization under Ministry of Water Resources, River
Development and Ganga Rejuvenation (Government of India), is a premier R&D
institute in the country to undertake, aid, promote and coordinate basic,
applied and strategic research on all aspects of hydrology and water resources
development. The Institute has its headquarters at Roorkee (Uttarakhand).
Ans 10: D
 The Ministry of Civil Aviation is responsible for formulation of national
policies and programmes for the development and regulation of the civil
aviation sector in the country. It is responsible for the administration of
the Air craft Act, 1934, Aircraft Rules, 1937 and various other
legislations pertaining to the aviation sector in the country.
 The Airports Economic Regulatory Authority (AERA) is a statutory body
constituted under the Airports Economic Regulatory Authority of India
Act. 2008 in 2009 with its head office at Delhi.
 Bureau of Civil Aviation Security (BCAS) has responsibility to coordinate,
monitor, inspect and train personnel in Civil Aviation Security matters.
LFB IAS Join Our Telegram @ lfbias
8888 024 021 Page 26
Ans 11: A

In pursuance of Article 17 of the Constitution of India, the Untouchability


(Offences) Act, 1955 was enacted. Subsequently, it was amended and renamed
in the year 1976 as the "Protection of CivilRights Act, 1955". Rules under this
Act, viz ―The Protection of Civil Rights Rules, 1977‖ were notified in 1977. The
Act extends to the whole of India and provides punishment for the practice of
untouchability. It is implemented by the respective state governments and
union territory administrations.

Scheduled Castes and the Scheduled Tribes (Prevention of Atrocities) Act

The Scheduled Castes and the Scheduled Tribes (Prevention of Atrocities) Act,
1989 (The PoA Act)came into force in 1990. This legislation aims at preventing
commission of offences by persons other than scheduled castes and scheduled
tribes against scheduled castes and scheduled tribes. Comprehensive Rules
under this Act, titled ―Scheduled Castes and the Scheduled Tribes(Prevention
of Atrocities) Rules, 1995 were notified in the year 1995, which, inter-alia,
provide norms for relief and rehabilitation. These Rules had not been amended
thereafter.

Ans 12: D

The Fifth Schedule under Article 244 (1) of Constitution defines ―Scheduled
Areas‖ as such areas as the President may by Order declare to be Scheduled
Areas after consultation with the Governor of the state. The Sixth Schedule
under Article 244 (2) of the Constitution relates to those areas in the states of
Assam, Meghalaya, Tripura and Mizoram which are declared as ―Tribal Areas‖
and provides for District Councils and/or Regional Councils for such Areas.
These Councils have been conferred with wide ranging legislative, judicial and
executive powers. The Fifth Schedule Areas: The criteria for declaring any area
as a ―Scheduled Area‖ under the Fifth Schedule are: (a) preponderance of tribal
population, (b) compactness and reasonable size of the area, (c) a viable
administrative entity such as a district, block or taluk, and (d) economic
backwardness of the area as compared to neighboring areas. The specification
of ―Scheduled Areas‖ in relation to a state is done by a notified Order of the
President, after consultation with the state governments concerned. The same
applies for altering, increasing, decreasing, incorporating new areas, or
rescinding any orders relating to ―Scheduled Areas‖.

LFB IAS Join Our Telegram @ lfbias


8888 024 021 Page 27
Ans 13: C

The advantages of Scheduled Areas are that:

a) The Governor of a state, which has Scheduled Areas, is empowered to


make regulations in respect of the following:
I. Prohibit or restrict transfer of land from tribal people;
II. Regulate the business of money lending to the members of Scheduled
Tribes. In making any such regulation, the Governor may repeal or
amend any Act of Parliament or of the Legislature of the state which is
applicable to the area in question.
b) The Governor may be through public notification direct that any
particular Act of Parliament or of the Legislature of the state, shall not
apply to a Scheduled Area or any part thereof in the state or shall apply
to such area subject to such exceptions and modifications as he may
specify.
c) the Governor of a state having Scheduled Areas therein, shall annually,
or whenever so required by the President of India, make are port to the
President regarding the administration of the Scheduled Areas in that
state and the executive power of the Union shall extend to the giving of
directions to the State as to the administration of the said area.
d) Tribes Advisory Council (TAC) shall be established in states having
Scheduled Areas. The role of TAC is to advise the state government on
matters pertaining to the welfare and advancement of the scheduled
tribes in the state as may be referred to it by the Governor. The TAC will
consist of not more than twenty members of whom about 3/4 are from
STMLAs. The TAC may also be established in any state having scheduled
tribes but not Scheduled Areas on the direction of the President of India
e) The Provisions of the Panchayats (Extension to Scheduled Areas) Act,
1996 (PESA), vide which the provisions of Panchayats, contained in Part
IX of the Constitution, were extended to Scheduled Areas, also contain
special provisions for the benefit of Scheduled Tribes.

Ans 14: B

The present Tribal Sub Plan (TSP) strategy was initially developed by an Expert
Committee set up by the Ministry of Education and Social Welfare in 1972 for
rapid socio-economic development of tribal people and was adopted for the first
time in the Fifth Five Year Plan. The TSP strategy, with some modifications,
continues till this day and the salient features with respect to TSP for states,
are: The funds provided under the Tribal Sub Plan of the state have to be at
LFB IAS Join Our Telegram @ lfbias
8888 024 021 Page 28
least equal in proportion to the ST population of each state or UT; tribal‘s and
tribal areas of a state or union territories are given benefits under the TSP, in
addition to what percolates from the overall Plan of a state/UT; The Sub-Plan
should;

a) Identify the problems and need of tribal people and critical gaps in
their development;
b) Identify all available resources for TSP;
c) Prepare a broad policy framework for development;
d) Define a suitable administrative strategy for its implementation; and
e) Specify the mechanism for monitoring and evaluation.

Ans 15: C

ICDS System

MWCD implementing International Development Association (IDA) assisted in


162 high burden districts of 8 states in the country covering 3.68 lakh
Anganwadi Centres with the following project development objective: (i) to
strengthen the Integrated Child Development Services (ICDS) policy framework,
systems and capacities, and facilitate community engagement, to ensure
greater focus on children under three years of age; (ii) to strengthen convergent
actions for improved nutrition outcomes. One of the key activities in ICDS
Systems Strengthening and Nutrition Improvement Project(ISSNIP) is
Information and Communication Technology enabled Real Time Monitoring
(ICT-RTM)of ICDS. It intends to leverage ICT to set up a real time monitoring
system for improving the service delivery mechanism and ensuring better
supervision of ICDS Scheme by deploying a mobile solution driven by a
customized ICDS-Common Application Software (ICDS-CAS) at the Anganwadi
Centres in selected States. A memorandum of co-operation was signed with Bill
and Melinda Gates Foundation (BMGF) to support the design, development and
deployment of Common Application Software (CAS) to drive the IT enabled Real
Time Monitoring of ICDS and Support System. Accordingly, a customized
Common Application software (ICDS-CAS) has been developed by BMGF.

Ans 16: A

Rajiv Gandhi Khel Ratna Award

The scheme was launched in the year 1991-92 with the objective of honouring
sportspersons to enhance their general status and to give them greater dignity
and place of honour in society. Under this scheme, an amount of ₹ 7.5 lakh is
LFB IAS Join Our Telegram @ lfbias
8888 024 021 Page 29
given as award for the most spectacular and outstanding performance by a
sportsperson over a period of four years immediately preceding the year in
which award is to be given.

Arjuna Award

The award was instituted in 1961. To be eligible for the award, a sportsperson
should have not only good performance consistently for the previous four years
at the international level with excellence for the year for which award is
recommended, but should have also shown qualities of leadership,
sportsmanship and a sense of discipline. The awardee is given a statuette, a
certificate, ceremonial dress and a cash award of five lakh. Not more than
fifteen awards are given every year.

Rashtriya Khel Protsahan Puraskar

With a view to recognizing the contribution made to sports development by


entities other than sports persons and coaches, Government has instituted a
new award titled ‗Rashtriya Khel Protsahan Puraskar‘ from 2009, which has
four categories, namely, identification and nurturing of budding/young talent,
encouragement to sports through Corporate Social Responsibility, employment
of sportspersons and Sports Welfare Measures and Sports for Development.
The award consists of a citation and a trophy in each of the above mentioned
categories. There is no cash award.

Ans 17: A

Innovation in science pursuit for inspired research (INSPIRE) is an ongoing


initiative for attraction of talent for science and research. About 3.67 lakh
students in the age group of 10-15 have been provided INSPIRE awards during
past 3 years. Science camps of 5 days duration are organised to provide
opportunity to class XI students pursuing science to interact with the science
icons from India and abroad including Nobel Laureates to experience the joy of
innovations. Around 40,000 students have been awarded scholarship for
higher education during last three years. About 3,000 students, who are first
rank holders at Masters level and also INSPIRE scholars completed their
Masters degree with securing 65 per cent marks at the MSc level, pursuing
doctoral degree are receiving INSPIRE fellowship during last three years.
INSPIRE faculty awards are provided to post-doctoral researchers in the age
group of 27-32 years through contractual and tenure positions for 5 years in

LFB IAS Join Our Telegram @ lfbias


8888 024 021 Page 30
basic and applied sciences. Faculty awards have been given to 637 candidates
awarded in the last three years.

Ans 18: C

The Council of Scientific & Industrial Research, (CSIR) constituted in 1942 is


an autonomous body which is known for its cutting edge R&D knowledge base
in diverse S&T areas. Having pan-India presence, CSIR has a dynamic network
of 38 national laboratories, 39 outreach centres, 3 innovation complexes and 5
units. Its R&D expertise and experience is embodied in about 4000 active
scientists supported by about 7000 scientific and technical personnel. It covers
a wide spectrum of science and technology—from radio and space physics,
oceanography, earth sciences, geophysics, chemicals, drugs, genomics,
biotechnology and nanotechnology to mining, aeronautics, instrumentation,
environmental engineering and information technology. It provides significant
technological intervention in many areas with regard to societal efforts which
include environment, health, drinking water, food, housing, energy, leather,
and farm and non-farm sectors. CSIR is the nation‘s custodian for
measurement standards of mass, distance, time, temperature, current, etc.
CSIR has created and is the custodian of Traditional Knowledge Digital Library
(TKDL) which is a powerful weapon against unethical commercial exploitation
of Indian traditional knowledge. It maintains Microbial Type Culture Collection
(MTCC) and Gene Bank. Pioneer of India‘s intellectual property movement,
CSIR today is strengthening its patent portfolio to carve out global niches for
the country in select technology domains.

Ans 19: C

The Heavy Water Board has contributed successfully to the first stage of
Nuclear Power Programme by producing heavy water for all Pressurised Heavy
Water Reactors in a cost effective manner enabling the department to provide
nuclear power at an affordable cost to common man. Presently, the Board is
not only self-sufficient in meeting the domestic demand of heavy water, but is
also geared upto supply heavy water for the future PHWRs and AHWRs as per
the envisioned nuclear power programme of DAE. Over a period of time, HWB
has emerged as the largest global producer and a trusted supplier of this
strategic material.

HWB has made major progress on its extended mandate in the field of
development, demonstration and deployment of technologies for in-core and
out of core material inputs for Indian Nuclear Power Programme, required at

LFB IAS Join Our Telegram @ lfbias


8888 024 021 Page 31
the front-end as well as back-end of nuclear fuel cycle mainly centering around
three broad areas such as solvents for front and back end hydrometallurgical
operations of nuclear fuel cycle; demonstrating solvent extraction technology
for extraction of various rare materials and producing stable isotopes like 10B
for Fast Breeder Reactors and 18O for societal applications.

Ans 20: C

Space activities in the country were initiated with the setting up of Indian
National Committee for Space Research (INCOSPAR) in 1962. In the same year,
work on Thumba Equatorial Rocket Launching Station (TERLS) near
Thiruvananthapuram was also started. Indian Space Research Organisation
(ISRO) was established in August, 1969. The Government of India constituted
the space commission and established the Department of Space (DOS) in 1972
and brought ISRO under DOS in1972. Space Commission formulates the
policies and oversees the implementation of the Indian space programme to
promote the development and application of space science and technology for
the socioeconomic benefit of the country. DOS implements these programmes
through, mainly, Indian Space Research Organisation (ISRO), Physical
Research Laboratory (PRL), National Atmospheric Research Laboratory (NARL),
North Eastern-Space Applications Centre (NE-SAC) and Semi-Conductor
Laboratory (SCL). Antrix Corporation, established in 1992 as a government
ownedcompany, markets the space products and services.The establishment of
space systems and their applications are coordinated by the national
levelcommittees, namely, INSAT Coordination Committee (ICC), Planning
Committee on National NaturalResources Management System (PC-NNRMS)
and Advisory Committee for Space Sciences(ADCOS).

Ans 21: B

The Press Information Bureau (PIB) is the nodal agency of the Government of
India to disseminate information to the print and electronic media on
government policies, programmes, initiatives and achievements. It functions as
an interface between the Government and the media and also provides
feedback to the Government on people‘s reaction as reflected in the media.

Prime Minister‘s Unit

PIB has a dedicated unit for the publicity and media support to the Prime
Minister‘s office. This unit functions all 365 days of the year on 24X7 basis.
The PM Unit compiles a number of reports on all days including holidays for

LFB IAS Join Our Telegram @ lfbias


8888 024 021 Page 32
PMO, Cabinet Secretariat and other senior officials of PMO. The Unit reaches
out extensively to regional/branch offices for ensuring nation-wide publicity
efforts, and gathering feedback for perusal of PMO.

Ans 22: A

Mahatma Gandhi National Rural Employment Guarantee Act (MGNREGA) has


come a long way since its inception, and has become a lifeline to millions. The
Act aims at enhancing livelihood security of households in rural areas of the
country by providing not less than one hundred days of guaranteed wage
employment in a financial year to every household whose adult members
volunteer to do unskilled manual work. Social inclusion, gender parity, social
security and equitable growth are the founding pillars of Mahatma Gandhi
NREGA.

The objectives of the scheme are: providing upto 100 days of unskilled manual
work in a financial year to every household in rural areas as per demand
resulting in creation of productive assets of prescribed quality and durability;
enhance livelihood security of the rural poor by generating wage employment
opportunities in works that develop the infrastructure base of the area
concerned; strengthening and securing the livelihood resource base of the rural
poor; ensure empowerment to women; stimulate the local economy by
providing a safety net to rural poor by proactively ensure social inclusion and
strengthening grass-root democratic institutions.

The government has been working towards bringing positive changes in the
programme with new initiatives. Several steps have been taken to strengthen
the overall implementation of MGNREGS within the overall framework of
MGNREG Act, 2005. Besides ensuring a high budget allocation of meet the
objectives of the Act, measures like electronic fund management system
(eFMS), Aadhaar seeding, geo tagging of assets and strengthening of Social
Audit System are some steps towards bringing in more transparency and
accountability in the programme implementation.

Ans 23: A

Cost of production (CoP) is one of the important factors in the determination of


MSP of mandated crops. Besides cost, the Commission considers other
important factors such as demand and supply, price trend in the domestic and
international markets, inter-crop price parity, terms of trade between
agricultural and non-agricultural sectors and the likely impact of MSPs on

LFB IAS Join Our Telegram @ lfbias


8888 024 021 Page 33
consumers, in addition to ensuring rational utilization of natural resources like
land and water. Thus, pricing policy is rooted not in ‗cost plus‘ approach,
though cost is an important determinant of MSP.

Ans 24: D

Swadesh Darshan scheme has a vision to develop theme based tourist circuits
on the principles of high tourist value, competitiveness and sustainability in an
integrated manner by synergizing efforts to focus on needs and concerns of all
stakeholders to enrich tourist experience and enhance employment
opportunities. Under the scheme thirteen thematic circuits have been identified
for development, namely: North-East India Circuit, Buddhist Circuit,
Himalayan Circuit, Coastal Circuit, Krishna Circuit, Desert Circuit, Tribal
Circuit, Eco Circuit, Wildlife Circuit, Rural Circuit, Spiritual Circuit, Ramayana
Circuit and Heritage Circuit.

Under PRASAD scheme, 25 sites of religious significance have been identified


for development namely Amaravati (Andhra Pradesh), Amritsar (Punjab), Ajmer
(Rajasthan), Ayodhya (UttarPradesh), Badrinath (Uttarakhand), Dwarka
(Gujarat), Deoghar (Jharkhand), Belur (West Bengal),Gaya (Bihar) , Guruvayoor
(Kerala), Hazratbal (Jammu & Kashmir), Kamakhya (Assam),Kanchipuram
(Tamil Nadu), Katra (Jammu & Kashmir), Kedarnath (Uttarakhand), Mathura
(UttarPradesh), Patna (Bihar), Puri (Odisha), Srisailam (Andhra Pradesh),
Somnath (Gujarat), Tirupati(Andhra Pradesh), Trimbakeshwar (Maharashtra),
Ujjain (Madhya Pradesh), Varanasi (Uttar Pradesh)and Vellankani (Tamil
Nadu).

Ans 25: A

The Central Statistics Office (CSO), an attached office of the Ministry,


coordinates the statistical activities in the country and evolves statistical
standards. Its activities inter-alia include compilation of national accounts,
index of industrial production, consumer price indices for
urban/rural/combined, human development statistics including gender
statistics, conduct of annual survey of industries and economic census and
imparting training in official statistics. The CSO also assists in the development
of statistics in the states and union territories and disseminates energy
statistics, social and environment statistics and prepares the national
industrial classification.

Ans 26: B

LFB IAS Join Our Telegram @ lfbias


8888 024 021 Page 34
CSO compiles the Index of Industrial Production (IIP) using secondary data
received from source agencies in various ministries/ departments or their
attached/ subordinate offices. The present base year of IIP is 2004-05. IIP is
released every month in the form of Quick Estimates with a time-lag of 6weeks
as per the Special Data Dissemination Standard (SDDS) norms of 23 IMF.
Apart from breakup of the index for mining, manufacturing and electricity
sectors, the estimates are also simultaneously being released as per use-based
classification viz., basic goods, capital goods, intermediate goods, consumer
durables and non-durables. These estimates are revised subsequently on
receipt of updated production data from the 15 source agencies. The major
source of data for IIP is, however, the Department of Industrial Policy and
Promotion that supplies data for 268 out of 399 item groups with a weight of
45.6 per cent in overall IIP.

CSO started compiling Consumer Price Index (CPI) separately for rural, urban,
and combined sectors on monthly basis with Base Year (2010=100) for all India
and states/UTs from January 2011. It revised the Base Year 14 of the CPI from
2010=100 to 2012=100, incorporating many methodological improvements in
consonance with the international practices.

On the request of Department of Industrial Policy and Promotion (DIPP),


Ministry of Commerce and Industries, NSSO is facilitating the Office of the
Economic Adviser in price data collection for existing series as well as New
Series of WPI. Currently the data transmission is being done in respect of
3,813 units and 18,192 quotations spread throughout thecountry. The Base
year for the Existing Series of WPI is 2004-05.

Ans 27: D

The Members of Parliament Local Area Development Scheme (MPLADS) was


launched in 1993 to provide a mechanism for Members of Parliament to
recommend works of developmental nature for creation of durable community
assets and for provision of basic facilities including community infrastructure,
based on locally felt needs to be taken up in their constituencies/states.
Initially, the MPLADS was under the control of Ministry of Rural Development.
The subject relating to the MPLADS was transferred to the Ministry of Statistics
and Programme Implementation in 1994.

The funds released under the Scheme are non-lapsable, i.e. the entitlement of
funds not released ina particular year is carried forward to the subsequent

LFB IAS Join Our Telegram @ lfbias


8888 024 021 Page 35
years, subject to eligibility. At present, the annual entitlement per MP/
Constituency is ₹ 5 crore.

Under MPLADS, the role of the Members of Parliament is limited to recommend


works. Thereafter, it is the responsibility of the district authority to sanction,
execute and complete the works recommended by Members of Parliament
within the stipulated time period.

Ans 28: D

E-Sampark Database is developed to send messages and emails to public


representatives and government employees. This database has 1.90 crore email
addresses and over 82 crore mobile numbers. The count of mailers is over 301
crore for 563 campaigns.

E-Taal is a web portal for dissemination of e-transactions statistics of national


and state level e-governance projects including mission mode projects. It
receives transaction statistics from web based applications periodically on near
real time basis. E-Taal presents quick analysis of transaction counts in tabular
and graphical form to give quick view of transactions done by various e-
governance projects.

Ans 29: B

DRDO is headed by the Scientific Adviser to Raksha Mantri who is also the
Secretary, Department of Defence Research and Development and Director
General R and D (DGRandD).

The mandate has been widened to support national cyber security architecture
which includes testing capabilities, security solutions, networking systems and
cyber defence tools.

The DRDO ensures training to all cadres of personnel through training


institutes, like DIAT, Pune (for technical courses); ITM, Mussoorie (for
technomanagerial programmes) and Defence Laboratory, Jodhpur (for
technical, administrative and allied cadre).

Ans 30: C

Kasturba Gandhi Balika Vidyalaya (KGBV): KGBV are residential upper


primary schools for girls from SC, ST, OBC Muslim communities and BPL girls.
KGBVs are set up in educational backward blocks where schools are at great
distances and are a challenge to their security of girls. KGBVs reach out to

LFB IAS Join Our Telegram @ lfbias


8888 024 021 Page 36
adolescent girls who are unable to go to regular schools to out of school girls in
the 10+ age group who are unable to complete primary school and younger
girls of migratory populations in difficult areas of scattered habitations that do
not qualify for primary/upper primary schools. KGBVs provide for a minimum
reservation of 75 per cent seats for girls from SC/ST/OBC and minorities
communities and 25 per cent to girls from families that live below the poverty
line.3,600 KGBVs are functional in the States and 3,66,756 girls are enrolled
in them.

Ans 31: A

 Statement 1 is correct: The SDGs, also known as Global Goals, build on


the success of the Millennium Development Goals (MDGs) and aim to go
further to end all forms of poverty. They include the 17 Sustainable
Development Goals (SDGs) and 169 targets.
 Statement 2 is not correct: The new Goals are universal and apply to all
countries, whereas the MDGs were intended for action in developing
countries only.
 Statement 3 is not correct: The Sustainable Development Goals (SDGs) are
not legally binding.

Ans 32: B

 Statement 1 is not correct: AAY contemplates identification of poorest of


the poor families from amongst the BPL families covered under TPDS.
 Statement 2 is correct: The beneficiaries receive 35 kg. of foodgrains per
household per month at a highly subsidised rate of 2 per kg for wheat and
3 per kg for rice.

Ans 33: D

This Act applies-

(a) to any person, who is a Hindu by religion in any of its forms or


developments, including a Virashaiva, a Lingayat or a follower of the
Brahmo, Prarthana or Arya Samaj,
(b) to any person who is a Buddhist, Jaina or Sikh by religion, and
(c) to any other person who is not a Muslim, Christian, Parsi or Jew by
religion, unless it is proved that any such person would not have been
governed by the Hindu law or by any custom or usage as part of the law
in respect of any of the matters dealt with herein if this Act had not been
passed.
LFB IAS Join Our Telegram @ lfbias
8888 024 021 Page 37
Ans 34: D

 Statement 1 is not correct: The Administered Pricing Mechanism (APM)


was abolished from 2002, consequent to the deregulation of the oil sector
in India. The Government notified that pricing of all petroleum products
except PDS kerosene and domestic LPG, would be market determined.
 Statement 2 is not correct. In June 2006, based on the recommendations
of the Rangarajan Committee, the Government changed the pricing
mechanism for petrol and diesel from import parity to trade parity (trade
parity being the weighted average of import parity and export parity
prices in the ratio of 80:20) while the pricing of PDS kerosene and
domestic LPG continues on import parity basis.

Ans 35: A

 All revenues received by government, loans raised by it, and also its
receipts from recoveries of loans granted by it, form the 'Consolidated
Fund'. All expenditure of Government is incurred from the Consolidated
Fund and no amount can be withdrawn from the fund without
authorisation from Parliament.Hence statement 1 is correct.
 Besides the normal receipts and expenditure of government which relate
to the Consolidated Fund, certain other transactions enter government
accounts, in respect of which, Government acts more as a banker, for
example, transactions relating to provident funds, small savings
collections and other deposits, etc. The moneys thus received are kept in
the Public Account and the connected disbursements are also made from
there. Parliamentary authorisation for such payments from the Public
Account is, therefore, not required.Hence statement 2 is incorrect.
 Occasions may arise when Government may have to meet urgent
unforeseen expenditure pending authorisation from Parliament. The
Contingency Fund is an imprest placed at the disposal of the President to
incur such expenditure. Parliamentary approval for such expenditure
and for withdrawal of an equivalent amount from the Consolidated Fund
is subsequently obtained and the amount spent from Contingency Fund
is subsequently recouped to the Fund.Hence statement 3 is incorrect.

Ans 36: C

 Statement 1 is correct: In August 2015, the Central Government revised


the National Solar Mission target of Grid Connected Solar Power projects
from 20,000 MW by 2022 to 1,00,000 MW by 2022. The centre has
LFB IAS Join Our Telegram @ lfbias
8888 024 021 Page 38
envisaged to achieve the 1,00,000 MW target by setting up Distributed
Rooftop Solar Projects and Medium & Large Scale Solar Projects.
 Statement 2 is correct: Two categories under which the target will be
achieved are:
 Category-I: Harvest 40,000 MW solar energy by setting Rooftop Solar
panels.
 Category-II: Harvest further 60,000 MW solar energy through the
collective efforts like Scheme for Decentralized Generation of Solar
Energy Projects by Unemployed Youths & Farmers, through PSUs, Large
Large Private Sector/IPPs, Solar Energy Corporation of India (SECI),
Under State Policies and Ongoing programmes incl. past achievements.

Ans 37: C

DAE has been pursuing the following 3-stage Nuclear Power Programme:

 The first stage comprises setting up of Pressurized Heavy Water Reactors


(PHWRs) and associated fuel cycle facilities. PHWRs use natural uranium
as fuel and heavy water as moderator and coolant.
 The second stage envisages setting up of Fast Breeder Reactors (FBRs)
backed by reprocessing plants and plutonium-based fuel fabrication
plants. Plutonium is produced by irradiation of uranium-238.
 The third stage is based on the thorium-uranium-233 cycle. Uranium-
233 is obtained by irradiation of thorium.

Ans 38: A

 Statement 1 is correct: Its objective is to celebrate unity in diversity of


the nation and to promote the spirit of national integration through
engagement between all Indian States and Union Territories.
 Statement 2 is correct: The states will undertake a unique partnership
for one year which would be marked by cultural and student exchanges.
Students of a particular state would travel to another state to learn each
other's culture.
 Statement 3 is not correct: It is an initiative of the Ministry of Culture not
a joint initiative with Ministry of HRD.

Benefits of Ek Bharat Shreshtha Bharat:

LFB IAS Join Our Telegram @ lfbias


8888 024 021 Page 39
 Benefits of exchanges in areas of language, trade, culture, travel and
tourism
 Benefits of promoting one state to other state every year
 The programme also quit beneficial for the students who will travel over
their states to meet new people and experience new life

Ans 39: B

 A greenfield airport is a new airport which is constructed on an


undeveloped site. Hence it is not constrained by prior constructions.
 Brownfield airport is continuing the development of a existing airport.
 Cochin international airport is the world's first airport that draws 100%
power through solar energy

Ans 40: D

 The Indian Coast Guard was formally established on 18 August 1978 by


the Coast Guard Act, 1978 of the Parliament of India as an independent
Armed force of India. It operates under the Ministry of Defence.
 The Coast Guard works in close cooperation with the Indian Navy, the
Department of Fisheries, the Department of Revenue (Customs) and the
Central and State police forces.
 The duties and functions of Coast Guard as enunciated in the Coast
Guard Act includes
 Safety and protection of artificial islands and offshore terminals,
installations and devices in maritime zones;
 protection and assistance to fishermen at sea while in distress;
 preservation and protection of marine environment;
 prevention and control of marine pollution;
 assistance to customs and other authorities in anti-smuggling
operations and enforcing of enactments being in force in the maritime
zones and other matters, including measures for the safety of life and
property at sea and collection of scientific data

Ans 41: A

 The Tibetan Medicine system is known as Sowa Rigpa (Amchi). The


system of Sowa Rigpa is practiced in many parts of India viz. Ladakh,
Sikkim, Darjeeling, Mon Tawang and West Kameng regions of Arunachal
Pradesh, etc. The medicine has been recognized by Government of India
(GoI) by amending the Indian Medicine Central Council (IMCC) Act, 2010.
LFB IAS Join Our Telegram @ lfbias
8888 024 021 Page 40
 The National Institute of Sowa Rigpa has been established at Leh in
Jammu & Kashmir.

Ans 42: D

 FSB was established in 2009 under the aegis of G 20 by bringing


together the national authorities, standard setting bodies and
international financial institutions for addressing vulnerabilities and
developing and implementing strong regulatory, supervisory and other
policies in the interest of financial stability.Hence statements 1 and 2 are
correct.
 India is an active member of the FSB. Hence statement 3 is correct.
 It has three seats in its Plenary represented by Secretary (EA), Deputy
Governor- RBI and Chairman, SEBI. Regular interaction with FSB takes
place through meetings and periodic conference call.

Ans 43: D

Pradhan Mantri Krishi Sinchai Yojana (PMKSY) conceived in 2015 is an


umbrella scheme for coverage of more and more area under assured irrigation
as early as possible. Its major components are:

(i) Accelerated Irrigation Benefits Programme (AIBP) for major and


medium irrigation including National projects;
(ii) Har Khet Ko Pani which includes command area development and
water management (CAD&WM) works, surface minor irrigation,
irrigation through groundwater and repair, renovation and restoration
(RRR) of water bodies;
(iii) Per drop more crop for promotion of micro irrigation ; and
(iv) Watershed development for rain water harvesting, effective
management of the run-off water, prevention of soil erosion,
regeneration of natural vegetation and recharging of the ground water
table.

Hence, all the statements are correct.

Ans 44: A

 Prasar Bharati is a statutory autonomous body established under the


Prasar Bharati Act and came into existence on 23.11.1997. It is the
Public Service Broadcaster of the country. The objectives of public service
broadcasting are achieved in terms of Prasar Bharati Act through All

LFB IAS Join Our Telegram @ lfbias


8888 024 021 Page 41
India Radio and Doordarshan, which earlier were working as media units
under the Ministry of I&B.
 Press Council of India is not part of Prasar Bharati.

Ans 45: A

 Statement 1 is correct: At the time of presentation of the Annual


Financial Statement before Parliament, a Finance Bill is also presented
in fulfilment of the requirement of Article 110(1) (a) of the Constitution,
detailing the imposition, abolition, remission, alteration or regulation of
taxes proposed in the budget. A Finance Bill is a Money Bill as defined in
Article 110 of the Constitution.
 Statement 2 is not correct: The purpose of the ‗Vote on Account‘ is to
keep government functioning, pending voting of final supply. The Vote on
Account is obtained from Parliament through an Appropriation (Vote on
Account) Bill.

Ans 46: A

The Ministry of Corporate Affairs (MCA) administers the following Acts of the
Central Government:

(i) (i)Companies Act, 2013


(ii) Companies Act, 1956
(iii) Limited Liability Partnership Act, 2008
(v) The Competition Act, 2002 as amended by Competition (Amendment)
Act, 2009
(iv) The Insolvency & Bankruptcy Code, 2016
(v) The Chartered Accountants Act, 1949; (vii) The Cost and Works
Accounts Act, 1959
(vi) The Company Secretaries Act, 1980; (ix) Societies Registration Act,
1860
(vii) Indian Partnership Act, 1932 (in Centrally administered areas)
(viii) The Partnership Act, 1932
(ix) Companies (Donations to National Funds) Act, 1951.

The Indian trade portal was launched under the Ministry of Commerce and
Industry. The Bureau of Indian Standards Act, 2016 is administered by the
Ministry of Consumer Affairs, Food and Public Distribution.

Ans 47: A

LFB IAS Join Our Telegram @ lfbias


8888 024 021 Page 42
 Leap Second is added to give Earth the opportunity to catch up with the
atomic time. recently the world uses precision atomic clocks in many
applications like satellites, where time is kept by measuring the
movements of electrons in cesium atoms.
 Consequently, atomic time is constant, but the Earth's rotation slows by
about two thousandths of a second per day. So leap seconds are
essential to ensuring atomic time does not move away from time based
on the Earth's spin. If it isn't corrected, such a drift would result in
clocks showing the middle of the day occurring at night.
 Thus, it is not related to leap year though coincidently it was added in
2016, a leap year. Before 2016, it was added in 1972.

Ans 48: C

Statements 1 and 2 are correct: Central Ground Water Board was constituted
as Central Ground Water Authority (CGWA) under sub-Section (3) of Section 3
of the Environment (Protection) Act, 1986 for the purpose of regulation and
control of ground water management and development in the country.

Ans 49: D

 Innovation in Science Pursuit for the Inspired Research (INSPIRE), a


National programme implemented by the Ministry of Science and
Technology is for attracting talent amongst students to study science and
pursue career with research, through Scholarships, Fellowships and
Mentorship
 KIRAN: Knowledge Involvement in Research Advancement through
Nurturing is a flagship scheme launched by DST for women.
 It provides opportunity to women scientists to pursue research in
physical, mathematical, chemical and life sciences etc.
 Focuses on SnT solutions at grass root levels for social benefits
 IMPRINT India is a Pan-IIT and IISc joint initiative to develop a roadmap
for research to solve major engineering and technology challenges in ten
technology domains relevant to India.

Ans 50: D

All options are correct.

Some important central laws under MoLE

- The Minimum Wages Act, 1948


LFB IAS Join Our Telegram @ lfbias
8888 024 021 Page 43
- The Payment of Wages Act, 1936
- The Payment of Bonus Act, 1965
- The Equal Remuneration Act, 1976
- The Trade Unions Act, 1926
- The Mines Act, 1952
- The Contract Labour (Regulation and Abolition) Act,1970
- The Bonded Labour System (Abolition) Act, 1976
- The Factories Act, 1948
- The Child Labour (Prohibition and Regulation) Act, 1986
- The Employment Exchange (Compulsory Notification of Vacancies) Act,
1959
- The Employees Provident Funds and Miscellaneous Provisions Act, 1952
- The Maternity Benefit Act, 1961

LFB IAS Join Our Telegram @ lfbias


8888 024 021 Page 44
If you think of UPSC then It’s

A dedicated institute in
MAharashtra for upsc only

Available UPSC Courses under


the guidance of Amol Dashpute Sir

UPSC- Integrated Upgraded foundation


Batch (18th June) Batch( 21st June)

Special Weekend Mains Success programme


Batch ( 25th June) Test Series ( 11th June)

UPSC- PRELIMS BATCH Interview Success


Programme
( 15th Jan)

Prelims Success programme


Test Series ( Nov. 18)

---------------Contact------------------
Khajina Vihir Chowk, Sadashiv Peth,
PUNE
8888 024 021
LFB IAS
8888 024 021 7888 232 233
Join Our Telegram @ lfbias
Page 45
Ans 51: C

The Financial Sector Assessment Programme is a joint programme of the


International Monetary Fund and the World Bank. In September 2010, IMF
made it mandatory for 25 jurisdictions (including India) with systemically
important financial sectors, to undergo financial stability assessments under
the FSAP every five years based on the size and inter-connectedness of their
financial sectors. It is reviewed periodically to make sure it reflects
developments in the global financial system.

Ans 52: C

 Genetic modification of disease causing mosquitoes has been suggested as


a method to control the spread of vector born diseases like Dengue,
Chikungunya, Malaria.
 The technology is conceptually simple. A gene is inserted into male Aedes
aegypti mosquitoes that transmit the viruses causing dengue,
chikungunya and zika fever. This gene produces a protein that can switch
off the activity of other genes key to the insect's survival. Adult males
carrying the self-destruct gene are, then released in large numbers in
endemic localities to mate with normal female Aedes aegypti mosquitoes
and produce offspring that cannot survive beyond larval stage. Hence,
statements 1 and 2 are correct.
 It is important to note that the lethal gene here is expressed only in the
offspring. The adult male is fed an antidote - the antibiotic tetracycline - to
prevent the self-destruct mechanism from operating, so as to ensure its
own survival and mating with female mosquitoes. In a matter of weeks, the
mosquito population drops drastically, as the offspring inheriting the gene
without the antidote die.
 The above genetically modified (GM) mosquito route to control dengue or
zika has been approved in countries from Brazil and the US to Cayman
Islands and Panama.

Ans 53: C

Statement 1 is not correct. The Minimum Wages Act, 1948 was enacted to
safeguard the interests of the workers mostly in the unorganized sector. The
categories of employees include unskilled (ex. Chowkidar, cleaner office peon
etc), semi-skilled /unskilled supervisory (ex. Cook, canteen boy, telephone
attendant etc), skilled (ex. Blacksmith, tailor, carpenter, painter etc), clerical
(ex. Record keeper, store keeper, munshi, typist, librarian etc), and highly

LFB IAS Join Our Telegram @ lfbias


8888 024 021 Page 46
skilled workers (ex. Compounder, head electrician, accountant, winding engine
driver etc.)

Statement 2 is correct. Under the provision of the Act, both the Central and
state governments are the appropriate governments to fix, revise, review and
enforce the payment of minimum wages to workers in respect of scheduled
employments under their respective jurisdictions. The enforcement of the
Minimum Wages Act, 1948 is ensured at two levels. Statement 3 is correct. To
protect the minimum wages against inflation, the Central Government has
introduced variable Dearness Allowance (VDA) linked to Consumer Price Index.

Ans 54: B

 The Union Cabinet has given its ex-post facto approval for
implementation of One Rank One Pension (OROP) retrospectively with
effect from 1st July, 2014.
 OROP will provide ex-servicemen of same rank and same length of service
uniform pension regardless of date of the retirement.
 Armed forces personnel who retire voluntarily would be not covered under
OROP scheme. However, it will be applicable to personnel who have
already retired prematurely.

Ans 55: A

 The Diamond Triangle consists of the three Buddhist sites of Ratnagiri,


Udayagiri & Lalitgiri. Belonging to the Vajrayana sect of Buddhism,
which is popularly known as the Diamond Vehicle, and hence the name
Diamond Triangle.
 The discovery of a huge number of sculptures, images, antiquity,
fragments, stone tablets, potteries, coins, stone and terracotta tablets,
and many huge stupas cemented the fact that these three places
described by Xuanzang (Hsuan-tsang) are the ruins of a university
complex called Pushpagiri university which shared an equal status with
Nalanda and Vikramshila universities.

Ans 56: B

A nationwide sub-programme of Sarva Shiksha Abhiyaan named Padhe Bharat


Badhe Bharat (PBBB) was launched in 2014 to ensure quality at the
foundational years of schooling i.e., classes I and II.

LFB IAS Join Our Telegram @ lfbias


8888 024 021 Page 47
Through this programme it will be ensured that all children are able to read
with comprehension as well as basic numeracy skills. The programme
envisages to have dedicated teachers for classes I and II. It centers on capacity
building of teachers, organizing separate reading periods in daily school time-
table, maintaining a print rich environment, for reading through children‘s
literature in school libraries and reading corners in classes I and II.

Ans 57: D

 Statement 1 is correct: The Scheme aims at empowering adolescent girls


of 11-18 years by improvement of their nutritional and health status and
upgrading various skills like home skills, life skills and vocational skills.
Thus the primary aim is health and nutrition of girls.
 Statement 2 is correct: The monitoring and supervision mechanism set up
under the Integrated Child Development Services (ICDS) Scheme is used
for the scheme Sabla as well.
 Statement 3 is correct: It is an initiative of the Ministry of Women and
Child Development.

Ans 58: B

 Polar cyclones (also known as Arctic Cyclones) are large areas of low
pressure. Polar cyclones are usually 1,000 to 2,000 kilometers wide in
which the air is moving in a spiral counterclockwise fashion in the
northern hemisphere. The reason for the rotation is the same as tropical
cyclones, the Coriolis effect.
 Thus, Polar Vortex is a circulation pattern of cold air (high pressure) that
traps ozone gas .It is in this zone that ozone depletion occurs.They occur
in polar regions like Greenland, Siberia and Antarctica. Polar cyclones
can form in any time of the year, although summer polar cyclones are
usually weaker than the ones that form in the winter. Hence, statements
1 and 3 are correct.
 Unlike tropical cyclones, polar cyclones are usually stronger in winter
months. Hence, statement 2 is notcorrect.
 They occur in regions that are not densely populated.

Ans 59: C

MERCOSUR is a trading bloc in Latin America comprising Brazil, Argentina,


Uruguay and Paraguay. MERCOSUR was formed in 1991 with the objective of
facilitating the free movement of goods, services, capital and people among the

LFB IAS Join Our Telegram @ lfbias


8888 024 021 Page 48
four member countries. It is the third largest integrated market after the
European Union (EU), North American Free Trade Agreement (NAFTA). India
has signed PTA with it in 2004.

Ans 60: B

 The Yamuna, the Ramganga, the Ghaghra, the Gandak, the Kosi, the
Mahananda and the Sone are the important tributaries of the Ganga.
 Rivers Chambal and Betwa are important tributary of the Yamuna.

Ans 61: B

 GCC: Gulf Cooperation Council (GCC)is a political and economic alliance


of six Middle Eastern countries -Saudi Arabia, Kuwait, the United Arab
Emirates, Qatar, Bahrain, and Oman. The GCC was established in
Riyadh, Saudi Arabia, in May 1981. The purpose of the GCC is to achieve
unity among its members based on their common objectives and their
similar political and cultural identities, which are rooted in Islamic
beliefs. Presidency of the council rotates annually.
 OPEC: The Organization of the Petroleum Exporting Countries (OPEC) is
a permanent, intergovernmental Organization, created at the Baghdad
Conference on September 10 -14, 1960, by Iran, Iraq, Kuwait, Saudi
Arabia and Venezuela.
 In accordance with its Statute, the mission of the Organization of the
Petroleum Exporting Countries (OPEC) is to coordinate and unify the
petroleum policies of its Member Countries and ensure the stabilization
of oil markets in order to secure an efficient, economic and regular
supply of petroleum to consumers, a steady income to producers and a
fair return on capital for those investing in the petroleum industry.

Ans 62: A

 Article 51A(a) of the Indian Constitution- It shall be the duty of every


citizen of India to abide by the constitution and respect its ideals and
institutions, the National Flag and the National Anthem.
 Recently, SC bench refused to intervene in plea to frame a national policy
to promote and propagate the 'National Song' along with the National
Anthem and the National Flag. The bench noted that Article 51A(a) of the
Indian Constitution refers only to National Flag and National Anthem.

Ans 63: B

LFB IAS Join Our Telegram @ lfbias


8888 024 021 Page 49
 Statement 1 is not correct; It functions under the Ministry of Labour and
Employment.
 Statement 2 is correct: Funding pattern: The projects have been taken up
in the Central Sector. The entire funding is done by the Central
Government (Ministry of Labour & Employment). Funds are released
depending upon the progress of project activities.
 Statement 3 is correct: The Scheme seeks to adopt a sequential approach
with focus on rehabilitation of children working in hazardous
occupations & processes in the first instance.

Details:

 Objective of the Scheme:


 This is the major Central Sector Scheme for the rehabilitation of child
labour.
 The Scheme seeks to adopt a sequential approach with focus on
rehabilitation of children working in hazardous occupations & processes
in the first instance.
 Under the Scheme, survey of child labour engaged in hazardous
occupations & processes has been conducted.
 The identified children are to be withdrawn from these occupations &
processes and then put into special schools in order to enable them to be
mainstreamed into formal schooling system.
 Project Societies at the district level are fully funded for opening up of
special schools/Rehabilitation Centres for the rehabilitation of child
labour.
 The special schools/Rehabilitation Centres provide:
 Non-formal/bridge education
 Skilled/vocational training
 Mid Day Meal
 Stipend @ Rs.150/- per child per month.
 Health care facilities through a doctor appointed for a group of 20
schools.
 The Target group:
 The project societies are required to conduct survey to identify children
working in hazardous occupations and processes. These children will
then form the target group for the project society. Of the children
identified those in the age group 5-8 years will have to be mainstreamed
directly to formal educational system through the SSA. Working children

LFB IAS Join Our Telegram @ lfbias


8888 024 021 Page 50
in the age group of 9- 14 years will have to be rehabilitated through
NCLP schools established by the Project Society.

Ans 64: D

 Statement 1 is correct: The BSI is the apex research organisation under


the MoEF&CC for carrying out taxonomic and floristic studies on wild
plant resources of the country.
 Statement 2 is correct: It brings out an inventory of endangered plants in
the form of a publication titled 'Red Data Book'
 Statement 3 is correct: It was established in 1890 with headquarter at
Kolkata.

Ans 65: D

 UGC has the unique distinction of being the only grant-giving agency in
the country which has been vested with two responsibilities: that of
providing funds and that of coordination, determination and
maintenance of standards in institutions of higher education.
 The University Grants Commission is a statutory organization
established by an Act of Parliament in 1956 (UGC act, 1956)
 The UGC's mandate includes:
 Promoting and coordinating university education.
 Determining and maintaining standards of teaching, examination
and research in universities.
 Framing regulations on minimum standards of education.
 Monitoring developments in the field of collegiate and university
education; disbursing grants to the universities and colleges.
 Serving as a vital link between the Union and state governments and
institutions of higher learning.
 Advising the Central and State governments on the measures
necessary for improvement of university education

Ans 66: A

 'skyTran' a NASA product was in news for being bought by the Govt. of
Bihar, as a mode of transportation in the state, which will run on
magnetic levitation (maglev)

LFB IAS Join Our Telegram @ lfbias


8888 024 021 Page 51
 'skyTrans' would be pod like, hanging on wire (elevated magnetic track),
which would be six meter above the ground and its diameter would be
equal to that of electric pole.
 It will also have very low electricity consumption of around 10 Kv, that
too just for charging the pod
 skyTran would move at the speed of 120-140km/hr within the city, and
in case of intercity journey, it would move at the speed of 240km/hr,
reducing the transportation time to a great extent.

Ans 67: A

 Orbiting the ringed planet Saturn and its numerous moons, the Cassini
spacecraft (unmanned) has been and continues to be a keystone of
exploration of the Saturnian system and the properties of gaseous
planets in our solar system. Hence, statement 1 is correct.
 A joint endeavor of NASA, the European Space Agency, or ESA, and the
Italian Space Agency, Cassini launched in 1997 along with ESA's
Huygens probe. Hence, statement 2 is not correct.
 The spacecraft contributed to studies of Jupiter for six months in 2000
before reaching its destination, Saturn, in 2004 and starting a string of
flybys of Saturn's moons. That same year it released the Huygens probe
on Saturn's moon Titan to conduct a study of the moon's atmosphere
and surface composition. Now in its second extended mission, which
goes through 2017, Cassini will make the first observations of a complete
seasonal period for Saturn and its moons.

Ans 68: C

National waterway-4:

 Kakinada-Puducherry canal stretch (767km) along with Godavari River


stretch (171km) between (Bhadrachalam and Rajhamundry) and Krishna
River stretch(157km) between (Wazirabad and Vijaywada) is termed as
NW-4.
 Total length of NW-4 is 1095 km. A network of irrigation cum navigation
canal linking Chennai and Ennore Ports in Tamil Nadu with the
Kakinada Port,and MaÂchlipatnam Ports in Andhra Pradesh runs
through a distance of 618 kms. The Kakinad canal and Eluru canal and
Commamur canal which are irrigation cum navigation canal also
interlink the two major river systems of Godavari and Krishna. The

LFB IAS Join Our Telegram @ lfbias


8888 024 021 Page 52
Buckingham canal which interlinks the Commamur canal with Chennai
port runs through a distance of 315 Km is tidal.
 The East Coast Canal stretch along with Brahmani and Mahanadi delta
river system is defined as NW-5.

Ans 69: A

The State Emblem of India is an adaptation from the Sarnath Lion Capital of
Asoka.

The profile of the Lion Capital shows three lions mounted on the abacus with a
Dharma Chakra in the centre, a bull on the right and a galloping horse on the
left, and outlines of Dharma Chakras on the extreme right and left.

Ans 70: B

 Securities Appellate Tribunal (SAT) is established under Section 15 K of


the Securities and Exchange Board of India Act, 1992, to exercise the
jurisdiction, powers and authority conferred on the Tribunal by or under
the SEBI Act, 1992, or any other law for the time being in force.Hence,
statement 1 is correct.
 The Tribunal is not bound by procedure laid down by Code of Civil
Procedure but is guided by principles of natural justice and has powers
to regulate its own procedure, including the places at which it shall have
its sittings. Hence, statement 2 is not correct.
 Any person aggrieved by a decision/order of SAT may file an appeal to
Supreme Court. SAT is also empowered to review its own decisions.
Hence, statement 3 is correct.

Ans 71: A

Ancient Names Present Day States

Dakshin Kaushal Chattisgarh


Pragjyotisha/Kamrupa Assam
Gomanchala, Gopakapattam, Gopakapuri,Govapuri, Gomantak Goa
Kalinga Odisha
Brahmarshi Desh/Madhya Desha Uttar Pradesh

Ans 72: D

LFB IAS Join Our Telegram @ lfbias


8888 024 021 Page 53
 International Fund for Agricultural Development (IFAD) was set up in
1977 as the thirteenth specialized agency of the United Nations. Hence
statement 1 is correct.
 It is dedicated to eradicating poverty and hunger in rural areas of
developing countries. IFAD provides low interest loans and grants to
developing countries to finance innovative agricultural and rural
development programmes and projects. Hence statement 2 is correct.
 176 countries are members of the IFAD, and these are grouped into three
lists: List - A: Developed Countries; List - B: Oil Producing Countries;
and List - C : Developing Countries. India is in List – C and founder
member of IFAD and a key contributor among the member countries.
Hence statement 3 is correct.

Ans 73: C

 CSO is an attached office of the Ministry of Statistics and Programme


Implementation.Hence statement 1 is incorrect.
 It releases Consumer Price Indices (CPI) for All-India and states/union
territories separately for rural, urban and combined (rural plus urban)
for the purpose of temporal price comparison with effect from January,
2011.Hence statement 2 is correct.
 CSO has revised the Base Year from 2010=100 to 2012=100. The annual
inflation rates based on this CPI series are available since January,
2012.Hence statement 3 is incorrect.

Ans 74: D

 Pair 1 is matched correctly: Bhakra Dam is a concrete gravity dam


across the Satluj River and is near the border between Punjab and
Himachal Pradesh in northern India.
 Pair 2 is matched correctly: The Pong Dam, also known as the Beas
Dam, is an earth-fill embankment dam on the Beas River in the state of
Himachal Pradesh, India, just upstream of Talwara. The purpose of the
dam is water storage for irrigation and hydroelectric power generation.
 Pair 3 is matched correctly:The Ranjit Sagar Dam, also known as Thein
Dam, is part of the hydroelectric project constructed by the government
of Punjab. The dam is located on the Ravi River.

Ans 75: D

LFB IAS Join Our Telegram @ lfbias


8888 024 021 Page 54
India's foreign exchange reserves comprise foreign currency assets (FCAs), gold,
SDRs and reserve tranche position (RTP) in the IMF. The main purpose of
holding foreign exchange reserves is to make international payments and hedge
against exchange rate risks.

Ans 76: D

 Department of Official Language - Ministry of Home Affairs.


 Department of Pharmaceuticals - Ministry of Chemicals and Fertilizers.
 Department of Disinvestment - Ministry of Finance.
 Department of Food and Public Distribution - Ministry of Consumer
Welfare, Food and Public Distribution.
 Department of Space is not under any ministry.
 Department of Land Resources - Ministry of Rural Development

Ans 77: D

 NISHANT is a UAV developed by DRDO primarily for the Indian Army


with an endurance period of 4 hrs and 30 minutes. It is primarily tasked
with intelligence gathering over enemy territory and also for
reconnaissance, training, surveillance, target designation, artillery fire
correction, damage assessment, etc.
 TAPAS 201 (RUSTOM-II), India's indigenously developed long-endurance
combat-capable drone is a Medium Altitude Long Endurance (MALE)
UAV. It has an endurance of 24 hours and can conduct surveillance and
reconnaissance missions for the country's armed forces. The UAV can
also be used as an unmanned armed combat vehicle on the lines of the
US's Predator drone.
 AURA is an autonomous unmanned combat air vehicle (UCAV), being
developed by the DRDO for the Indian Air Force and Indian Navy. The
UCAV will be capable of releasing missiles, bombs and precision guided
munitions. The programme is in its project definition stage.
 LAKSHAYA is a Pilotless Target Aircraft (PTA).It is a reusable aerial target
system or an Indian Remotely piloted high speed target drone system
developed by the Aeronautical Development Establishment (ADE) of
DRDO for Indian Armed Forces.

Ans 78: D

All the pairs are correctly matched.

LFB IAS Join Our Telegram @ lfbias


8888 024 021 Page 55
 Rajasthan - Second largest producer of crude oil in India (location -
Western Rajasthan)
 Uranium is found in Khasi hills of Meghalaya.
 Jharkhand is one of the largest producer of coal in India.

Ans 79: D

All the statements given are correct.

 Central Adoption Resource Authority (CARA) is a statutory body of


Ministry of Women & Child Development, Government of India under
Juvenile Justice Act, 2015. It functions as the nodal body for adoption of
Indian children and is mandated to monitor and regulate in-country and
inter-country adoptions. CARA is designated as the Central Authority to
deal with inter-country adoptions in accordance with the provisions of
the Hague Convention on Inter-country Adoption, 1993, ratified by
Government of India in 2003.
 CARA primarily deals with adoption of orphan, abandoned and
surrendered children through its associated /recognised adoption
agencies.

Ans 80: A

 Kisan Credit Cards are offered to farmers in India in order to enable


them to access affordable credit for farmers. There is no component of
interest free loans under it. The scope of the KCC has been broadbased
to include term credit and consumption needs. The KCC Scheme has
since been simplified and converted into ATM enabled debit card with,
inter alia, facilities of one-time documentation. Statement 1 is correct
and statement 2 is not correct.
 Statement 3 is not correct. It is operational throughout the country. It is
implemented by Commercial Banks, Cooperative Banks and Regional
Rural Banks.

Ans 81: B

Some of the Organisations/institutions under Ministry of Culture:

 Tangible Culture Heritage


 Archaeological Survey of India
 National Museum
 National Gallery of Modern Art
LFB IAS Join Our Telegram @ lfbias
8888 024 021 Page 56
 National Council of Science Museums
 Gandhi Heritage Sites Mission
 National Culture Fund
 tangible cultural Heritage
 National School of Drama
 Indira Gandhi National Centre for the Arts
 Sahitya Akademi
 Lalit Kala Akademi
 Sangeet Natak Akademi
 The Centre for Cultural Resources and Training
 Kalakshetra Foundation

Ans 82: D

 Statement 1 is not correct: First Generation Biofuel: They are produced


directly from food crops. Crops such as wheat and sugar are the most
widely used feedstock
 Second Generation Biofuel: They are produced from marginal croplands
unsuitable for food production or non-food crops such as wood, organic
waste, food crop waste and specific biomass crops. For example-
Jatropha Thus, it overcomes over food vs fuel debate in first generation
biofuel.
 Third Generation Biofuel: It is based on improvements on the production
of biomass by taking advantage of specially engineered energy crops such
as algae as its energy source. The algae are cultured to act as a low-cost,
high-energy and entirely renewable feedstock. Algae will have the
potential to produce more energy per acre than conventional crops.
 Fourth Generation Biofuel: Fourth Generation Bio-fuels are aimed at
producing sustainable energy and also capturing and storing carbon
dioxide.
 Statement 2 is not correct: Biofuels are basically hydrocarbons thus
produce gases like CO2 etc while burning.

Ans 83: C

 The International Union of Pure and Applied Chemistry (IUPAC) is an


international federation of National Adhering Organizations that
represents chemists in individual countries
 IUPAC was established in 1919 as the successor of the International
Congress of Applied Chemistry for the advancement of chemistry.
LFB IAS Join Our Telegram @ lfbias
8888 024 021 Page 57
 IUPAC is best known for its works standardizing nomenclature in
chemistry.
 Some important work IUPAC has done in these fields includes
standardizing nucleotide base sequence code names; publishing books
for environmental scientists, chemists, and physicists; and improving
education in science.
 IUPAC is also known for standardizing the atomic weights of the
elements through one of its oldest standing committees, the Commission
on Isotopic Abundances and Atomic Weights.
 IUPAC is in news due to discovery of all the elements of 7th group of the
periodic table.

Ans 84: A

 Statement 1 is correct :Urea is the only fertilizer under statutory price


control and its import is restricted and permitted through State Trading
Enterprises (STEs), namely Metals and Mineral Trading Corporation of
India (MMTC), State Trading Corporation Limited (STC) and Indian
Potash Limited (IPL), under the Foreign Trade Policy of the Government.
 Statement 2 is not correct: Urea's import is restricted and permitted
through State Trading Enterprises (STEs). Import of fertilizers, other than
Urea, is free, commonly known as Open General License (OGL). Various
companies import these fertilizers as per their commercial judgment.
Muriate of Potash (MOP) is the only fertilizer, whose demand is fully met
through imports as there are no viable sources of MOP in the country.

Ans 85: A

 Jal Kranti Abhiyan was celebrated to consolidate water conservation and


management in the country through a holistic and integrated approach
involving all stakeholders, making it a mass movement.
 The objectives of Jal Kranti Abhiyan are :-
 Strengthening grass root involvement of all stakeholders including
Panchayati Raj Institutions and local bodies in the water security and
development schemes (e.g. Participatory Irrigation Management (PIM);
 Encouraging the adoption/utilization of traditional knowledge in
water resources conservation and its management;
 To utilize sector level expertise from different levels in government,
NGO's, citizens etc;
 Enhancing livelihood security through water security in rural areas.

LFB IAS Join Our Telegram @ lfbias


8888 024 021 Page 58
Ans 86: C

 Statement 1 is not correct: It is an intermediate range surface to surface


ballistic missile.
 Statement 2 is not correct: The nuclear-capable missile has a strike
range of 5,500km to 5, 800 km. Its range extends to entire Pakistan and
also northernmost parts of China thus adding to our defence
preparedness. It has been developed by DRDO.
 Statement 3 is correct: It can be transported and swiftly launched from
anywhere on land. It can even be launched from canisters. Also it can be
launched from a mobile platform, which gives the armed forces flexibility
to transport and fire it swiftly from anywhere they want.

Ans 87: D

 World Investment Report is published by UNCTAD (United Nations


Conference on Trade and Development)
 Global Wage Report is published by International Labour Organization.
 Global Environment Outlook is published by UNEP.

Ans 88: A

The port, situated on the bank of a river far away from the sea and connected
through that river with the sea, is called Riverine Port. Kolkata Port is the only
riverine major port in the country. It has a vast hinterland comprising the
entire Eastern India including West Bengal, Bihar, Jharkhand, U.P., M.P.,
Assam, North East hill states and the two landlocked neighbouring countries
namely, Nepal and Bhutan.

Ans 89: B

E-BRC (Electronic Bank Realization Certificate) project launched in 2012


created an integrated platform for receipt, processing and subsequent use of all
bank realization related information by exporters, Banks, Central and state
government departments. e-BRC was made mandatory in 2012. In a short span
of less than three years, it has brought down the cost of doing business for
Indian exporters and enhanced the productivity of banks, DGFT and many
other organizations. So far, 95 banks have transmitted more than 1.4 crore e-
BRCs.

Ans 90: D

LFB IAS Join Our Telegram @ lfbias


8888 024 021 Page 59
 Copper nanoparticles were incorporated in PVA films to develop active
antimicrobial films. They are also show great catalytic activities and can
be applied to biosensors and electrochemical sensors.
 Silver nanoparticles synthesized using mint extract were found to be
bactericidal against E. coli, S. aureus, B. cereus and P. aeruginosa.
(Introduction of silver into bacterial cells induces a high degree of
structural and morphological changes, which can lead to cell death) Due
to antibacterial properties of silver nanoparticles, multitude of appliances
such as air conditioners, washing machines, and refrigerators are fitted
with Silver nanoparticles. e.g. Samsung air conditioners.
 Gold nanoparticles are used in modern medical and biology studies.
Genomics, biosensorics, detection and photothermolysis of
microorganisms and cancer cells; the targeted delivery of drugs, in
almost all medical applications like diagnostics, therapy, prevention, and
hygiene.

Ans 91: B

 Statement 1 is correct: India is the largest producer as well as the


consumer of pulses in the world. These are the major source of protein in
a vegetarian diet. Major pulses that are grown in India are tur (arhar),
urad, moong, masur, peas and gram.
 Statement 2 is correct: Till 2013-14 under National Food Security
Mission, only 3 crops - rice, wheat and pulses were covered. Now, seven
crops viz. rice, wheat, pulses, jute, sugarcane, cotton and cereals have
been covered under the mission.
 Statement 3 is not correct : The Union Government constituted a high-
level committee headed by Chief Economic Adviser Arvind Subramanian
to tackle the shortage of pulses in India.
 N.K. Singh committee was set up in 2016 to review the Fiscal
Responsibility and Budget Management (FRBM) Act of 2003.

Ans 92: B

Some of the important identified mangrove sites along with states are as
follows:

 Odisha: Bhaitarkanika, Mahanadi, Subarnarekha, Chillka, Devi,


Dhamra
 Gujarat: Gulf of Kutch, Gulf of Khambat, Dumas-Ubhrat
 Tamil Nadu: Pichavaram, Muthupet, Ramnad, Pulicat, Kazhuveli
LFB IAS Join Our Telegram @ lfbias
8888 024 021 Page 60
 Karnataka: Karwar, Coondapor, Dakshin Kannada,Mangalore Forest
Division
 West Bengal: Sunderbans
 Andhra Pradesh: Coringa, East Godavari, Krishna
 Maharashtra: Achra-Ratnagiri, Devgarj-Vijay Dur, . Veldur, Kundalika-
Revdanda, Mumbai-Diva

Ans 93: D

Statement 1 is correct. PIB has a dedicated unit for the publicity and media
support to the Prime Minister's office. This unit functions all 365 days of the
year on 24X7 basis. The PM Unit compiles a number of reports on all days
including holidays for PMO, Cabinet Secretariat and other senior officials of
PMO.

About PIB:

 The Press Information Bureau (PIB) is the nodal agency of the


Government of India to disseminate information to the print and
electronic media on government policies, programmes, initiatives and
achievements. It functions as an interface between the Government and
the media and also provides feedback to the Government on people's
reaction as reflected in the media.
 Statement 2 is correct. PIB provides accreditation to media
representatives including foreign media at the Headquarters in New
Delhi. A fully on-line system of accreditation has been operationalised
from 2010 onwards to make the process prompt and efficient.
 Statement 3 is correct. One of the important functions of the Press
Information Bureau is to keep the Government informed of public
perception about government policies and programmes as reflected in
media. The feedback reports prepared by the PIB include inputs from the
national English and Hindi dailies published from the capital, inputs
from regional language newspapers as sent by the Regional/Branch
offices of the PIB, inputs from TV news channels, web media and
magazines. PIB Officers provided feedback to their respective Ministries
and Departments.

Ans 94: C

Skills Assessment Matrix for Vocational Advancement of Youth (SAMVAY) is a


credit framework which allows vertical and lateral mobility within vocational

LFB IAS Join Our Telegram @ lfbias


8888 024 021 Page 61
education system and between the current education systems. The strength of
this framework is the seamless integration of pursuit of academic knowledge
and practical vocational skills. Efforts like these will improve the employability
of our educated youth.

Ans 95: A

India is likely to purchase Japanese amphibious airplanes, the ShinMaywa US


2i. The aircraft will be helpful in humanitarian assistance and disaster relief
especially in Andaman & Nicobar Islands and in Lakshadweep, where no
regular landing strips exist currently. It is designed for search and rescue
operation not combat role.

Ans 96: A

Statement (a) is correct.

Employees' Provident Fund Organisation (EPFO) introduced a new system


called Unique Account Number (UAN) which helps employees manage their
provident fund accounts without any hassles. UAN is a unique 12-digit number
assigned to an employee by EPFO India. The EPFO UAN number will act as a
marker of identity for an employee till his/her retirement. In order to generate
a UAN number, an employee needs to be registered with EPFO services and
should contribute to the Employee Provident Fund Account. The major motive
behind the launch of UAN is to link multiple PF account numbers allotted to a
single employee by different organizations under one single account. In case an
employee is already registered with EPFO services and has a UAN number,
then he/she has to provide the same on joining a new organization so that the
new employer can link the new member Identification number of the
organization to his/her existing Universal Account Number. UAN can be
accessed by an employee by visiting the UAN member portal website.

Ans 97: D

 Resourcesat-2A is a remote sensing satellite which will provide


information on water bodies, farm lands, crop extent, forests, mineral
deposits, coasts, rural and urban spreads for the next five years. Hence,
statement 1 is correct.
 The satellite was placed into the Sun Synchronous Orbit at 825 km
height. Hence, statement 3 is correct.
 Remote sensing satellites are different from communication satellites.
Hence, statement 2 is not correct.
LFB IAS Join Our Telegram @ lfbias
8888 024 021 Page 62
LFB IAS Join Our Telegram @ lfbias
8888 024 021 Page 63
Ans 98: C

Islands in the IOR Parent Country

Socotra Yemen
Tromelin France
Cocos Australia
Christmas Australia
Reunion France
Glorioso France
Ashmore and Cartier Australia

Ans 99: B

 Ehang 184 is a passenger drone that debuted in 2016 and has the
capacity to fly 15Km for more than 20 minutes in a single charge.
 It is designed with full redundancy If one set of the power system are
operating abnormal, the vehicle can still operate a normal flight plan and
ensure the safety of the passenger together with the vehicle.
 It was designed to be a 100% with green technology, and is powered by
electricity only.

Ans 100 : A

 GRAPES-3 experiment (or Gamma Ray Astronomy PeV EnergieS phase-3)


is a special telescope-array established in Ooty to detect muons (an
elementary particle similar to the electron) from cosmic ray showers.
GRAPES-3 is designed to study cosmic rays with an array of air shower
detectors and a large area muon detector.
 The experiment has detected a surge in muon intensity correlated with a
weakening of the earth's magnetic field due to a solar storm that hit the
earth on June 22, 2015.
 An Indo-Japanese collaboration, this experiment is unique in that it can
be used to study solar storms and space weather at distances up to two
times the earth's radius, unlike satellite-based studies that can yield
information only about what is happening in their vicinity.

LFB IAS Join Our Telegram @ lfbias


8888 024 021 Page 64

You might also like